Surgical Principles Flashcards
2022 CSAT - Module 5:
A 10-year-old boy undergoes surgical repair of microtia of the right ear. During cartilage rib harvest, the
right thorax is damaged with visualization of the lung. After repair of the pleura, Valsalva maneuver is
performed with no evidence of an air leak. An intraoperative chest x-ray is negative for pneumothorax.
Several minutes later in the PACU, the patient becomes hypotensive and tachypneic, and his oxygen
saturation decreases to the mid-80s, despite use of a non-rebreather mask.Which of the following is the
most appropriate next step in management?
A) Draw arterial blood gas
B) Intubation
C) Needle decompression of the right chest
D) Open the chest incision
E) Portable chest x-ray study
The correct response is Option C.
The patient shows all the signs of tension pneumothorax, and although the precise etiology is unclear,
the patient requires decompression.
Intubation will not help relieve the tension and pressure, with decreased venous return jeopardizing
hemodynamic stability.
Immediate chest x-ray is inappropriate because of the time required.
Opening the chest incision is not a good option because it requires surgical equipment, general
anesthesia, and cannot be completed in a timely fashion.
Needle decompression at the second intercostal is the standard of care to decompress a tension
pneumothorax. After oxygen saturation and hemodynamics are stabilized, definitive treatment of
pneumothorax can be pursued. This would include placement of chest tube to low suction and serial
chest x-ray to monitor the progress of the lung inflation.
Arterial blood gas will not help make the diagnosis and potentially will delay the appropriate intervention.
Reference(s)
1. Sahn SA, Heffner JE. Spontaneous pneumothorax. N Engl J Med. 2000; 342:868.
2. Dotson K, Johnson LH. Pediatric spontaneous pneumothorax. Pediatr Emerg Care. 2012; 28:715.
3. Romo T, Baratelli R, Raunig H. Avoiding complications of microtia and otoplasty. Facial Plast Surg.
2012 Jun;28(3):333-9.
Which of the following procedures has the greatest risk of potential venothromboembolism (VTE)?
A) Abdominoplasty
B) Bilateral reduction mammaplasty
C) Implant-based calf augmentation
D) Liposuction of the trunk
The correct response is Option A.
There is level II evidence provided by Winocour et al in 2017 by querying the Cosmetassure database of
more than 129,000 patients that body procedures such as lower body lift and abdominoplasty have
higher risk than breast, liposuction, or facial procedures. More specific level II evidence about
abdominoplasty and venothromboembolism (VTE) was published in 2018 Keyes et al. after querying the
Internet Based Quality Assurance Program database, that BMI greater than 25 kg/m2 and age greater
than 40 were independent predictors of VTE risk. Most of the patients in this study who had VTE had preoperative
Caprini scores of 2 to 8, which would not typically make these patients recipients of
chemoprophylaxis against VTE.
Although operating in the area of the calf muscles seems like a good source of potential VTE, there is no reference to calf implants in these large database studies, and a PubMed search of VTE and calf
implants returns no literature.
For a generalized summary of risk stratification, consult the Pannucci et al. article.
Reference(s)
1. Pannucci CJ, Swistun L, MacDonald JK, et al. Individualized Venous Thromboembolism Risk
Stratification Using the 2005 Caprini Score to Identify the Benefits and Harms of Chemoprophylaxis in
Surgical Patients: A Meta-analysis. Ann Surg. 2017 Jun;265(6):1094-1103.
2. Winocour J, Gupta V, Kaoutzanis C, et al. Venous Thromboembolism in the Cosmetic Patient:
Analysis of 129,007 Patients. Aesthet Surg J. 2017 Mar 1;37(3):337-349.
3. Keyes GR, Singer R, Iverson RE, Nahai F. Incidence and Predictors of Venous Thromboembolism in
Abdominoplasty. Aesthet Surg J. 2018 Feb 17;38(2):162-173.
A 36-year-old health-care worker sustains a needle-stick injury from a hepatitis C–seropositive patient.
Immediate testing for anti-HCV antibodies and confirmatory immunoassays for HCV-RNA are performed.
Initial follow-up testing after exposure should be performed at which of the following time periods?
A) 1 week
B) 3 weeks
C) 6 weeks
D) 12 weeks
E) 24 weeks
The correct response is Option C.
It is recommended that follow-up retesting be done at 6 weeks, 3 months, and 6 months in known HCV
exposure cases. Tests at 1 or 3 weeks would possibly lead to false negative results. There is no
advantage in waiting beyond 6 weeks.
Reference(s)
1. Waljee J, Malay S, Chung K. Sharps Injuries: the Risks and Relevance to Plastic Surgeons. Plast
Reconstr Surg. 2013 Apr;131(4):784-91.
2. Pappas N, Lee DH. Hepatitis C and the Hand Surgeon: What You Should Know. Journal of Hand
Surgery. J Hand Surg Am. 2012 Aug;37(8):1711-3; quiz 1714.
A newborn male infant who is born at 36 weeks’ gestation via cesarean delivery has a large defect of the
anterior abdominal wall. Examination shows matted bowelloops coming through the defect lateral to the
umbilical cord. No other abnormalities are noted. Which of the following associated findings is/are most
likely?
A) Abnormal karyotype
B) Constriction rings with limb and digital amputations
C) Elevated maternal serum alpha fetoprotein (MSAFP)
D) Hypoglycemia, macrosomia, and macroglossia
E) Translucent membrane covering bowel
The correct response is Option C.
Omphalocele (OC) and gastroschisis (GS) represent the two most common congenital abdominal wall
defects, with a prevalence of approximately 3 to 4 per 10,000 live births/fetal deaths/stillbirths/pregnancy
terminations each. Precise pathoetiologies are unclear, but developmental pathways and characteristics
at the time of birth are notably distinct. OC is characteristically a midline partial-thickness abdominal wall
defect covered by a membrane of amnion and peritoneum occurring within the umbilical ring and
containing abdominal contents. GS is characteristically a full-thickness, paraumbilical abdominal wall
defect associated with eviscerated bowel.
Both OC and GS are associated with elevated maternal serum alpha fetoprotein (MSAFP). For
comparison, MSAFP values average twice that recorded in pregnancies with open spina bifida, and
similar to values recorded with anencephaly. An elevated MSAFP is an indication for thorough ultrasound
examination of the fetus for anatomical abnormalities.
Multiple chromosomal abnormalities have been associated with at least 60% OC cases, including trisomy
-18, -13, -21, Turner syndrome, and triploidy. By contrast, GS is associated with abnormal karyotype in
about 1% of cases, usually in the setting of other congenital abnormalities.
The definite treatment of both OC and GS is surgical once optimal resuscitation is achieved. Primary
closure is associated with better survival rates if it can be achieved without compromise of intestinal
blood flow or other hemodynamic or respiratory embarrassment. Large defects are frequently managed
with temporary abdominal silos which are gradually reduced over the course of days to weeks in a form
of visceral tissue expansion followed by delayed abdominal wall closure. The long-term outcome in
isolated cases of OC and GS are generally good, although they can be associated with gut motility
impairment, gastroesophageal reflux, ventral hernias, and late obstructive episodes.
Constriction rings with limb and digital amputations are found in amniotic band sequence but are not
characteristic of OC or GS. GS is not characteristically associated with hypoglycemia, macrosomia, or
macroglossia.
Reference(s)
1. Stephenson CD, Lockwood CJ, MacKenzie AP. Omphalocele. Wilkins-Haug L &Levine D, ed.
UpToDate. Waltham, MA: UpToDate Inc. http://www.uptodate.com. Accessed January 1, 2018.
Hyperbaric oxygen therapy (HBOT) is most appropriate for a patient with which of the following
conditions?
A) Acute osteomyelitis of the tibia
B) Anaerobic necrotizing soft-tissue infection
C) Chemical burn because of lye exposure
D) Stevens-Johnson syndrome
E) Wagner grade 2 diabetic foot ulcer
The correct response is Option B.
Hyperbaric oxygen therapy (HBOT) is an accepted adjunct to surgical debridement, appropriate antibiotic
therapy, and indicated critical care measures for necrotizing soft-tissue infections such as necrotizing
fasciitis and Fournier gangrene. The increased oxygen delivery of HBOT improves leukocyte function and
can enhance penetration of certain antibiotics such as aminoglycosides. The clinical effects include
slowing of the progress of the infection and decreased risk of both amputation and mortality.
There is not adequate evidence to justify HBOT in diabetic foot ulcers (DFUs) with Wagner grade 2
(extension to bone, tendon, or capsule) or less. However, there is moderate evidence to suggest benefit
in DFUs with Wagner grade 3 (deep ulcer with osteomyelitis or abscess) or greater.
HBOT may be indicated in the treatment of chronic osteomyelitis but not in the acute setting. It plays no
role in the treatment of Stevens Johnson syndrome, and may be beneficial in acute thermal burns but is
not indicated for chemical burns.
The complete list of approved indications for HBOT, as determined by the Undersea and Hyperbaric
Medical Society and the U.S. Food and Drug Administration, includes the following:
1. Air or gas embolism
2. Carbon monoxide poisoning
3. Clostridial myositis and myonecrosis (gas gangrene)
4. Crush injury, compartment syndrome, and other acute traumatic ischemias
5. Decompression sickness
6. Arterial insufficiency
7. Severe anemia
8. Intracranial abscess
9. Necrotizing soft-tissue infections
10. Refractory osteomyelitis
11. Delayed radiation injury (soft tissue and bony necrosis)
12. Compromised grafts and flaps
13. Acute thermal burn injury
14. Idiopathic sudden sensorineural hearing loss
Reference(s)
1. Fife CE, Eckert KA, Carter MJ. An update on the appropriate role for hyperbaric oxygen: indications
and evidence. Plast Reconstr Surg. 2016 Sep; 138(3 Suppl):107S-16S.
2. Weaver LK, ed. Hyperbaric Oxygen Therapy Indications, 13 th edition. Durham, NC. 2014:1-240.
A 28-year-old postpartum woman comes to the office for evaluation of breast asymmetry with pain and
enlargement of the right breast for 2 months. Medical history includes augmentation mammaplasty 4
years ago. She denies fever or chills. She was previously breast-feeding but stopped this 1 month ago.
Physical examination showsthe right breast is significantly larger than the left breast. A well-healed periareolar
incision is present and no evidence of infection is noted. Ultrasound shows a complex cyst, which
yields 150 cc of milky fluid. A drain is placed. The most appropriate next step is administrationof which of
the following medications?
A) Bromocriptine
B) Cephalexin
C) Fluconazole
D) Prolactin
E) Trimethoprim-sulfamethoxazole
The correct response is Option A.
This postpartum patient is presenting with a symptomatic galactocele after breast-feeding. Galactoceles
are benign breast cysts containing milk. They typically occur in women of childbearing age in the setting
of active lactation, recent pregnancy, or the use of hormonal medications such as oral
contraceptives. The galactocele is thought to occur from ductal obstruction. Although the presence of a
breast implant and the respective pocket placement is unknown to have an effect on the development of
galactoceles, there is some thought that peri-areolar incisions may contribute to the ductal obstruction.
There are, however, documented cases of post-augmentation galactocele without peri-areolar incisions.
Treatment for a galactocele is typically medical with the initiation of oral bromocriptine. Bromocriptine is a
dopamine receptor agonist and causes inhibition of prolactin secretion, which is the primary hormone
responsible for milk production. Dosage is titrated to effect. Incision and drainage of the cyst, particularly
in the setting of implants, is often performed as well to rule out the possibility of infection.
Cephalexin and trimethoprim-sulfamethoxazole are antibiotics and are not indicated in this case because
there is no active infection. Fluconazole is indicated for the treatment of fungal infections. Prolactin would
actually stimulate milk production and would worsen the patient’s symptoms.
Reference(s)
1. Rosique RG, Rosique MJ, Peretti JP. Postaugmentation Galactocele Without Periareolar Incision and
8 Years after Pregnancy. Plast Reconstr Surg Glob Open. 2016; 4(3): e644.
2. Tung A, Carr N. Postaugmentation Galactocele: A Case Report and Review of the Literature. Ann
Plast Surg. 2011; 67(6): 668-670.
A 70-year-old man is in the recovery room after undergoing radial forearm free flap reconstruction for
squamous cell carcinoma of the tongue. He has a 30-pack year history of smoking. The patient is
ventilated witha tracheostomy tube. Two days postoperatively, sedation is turned off for an hour with the
goal of weaning the patient off mechanical ventilation. He becomes agitated and delirious. Which of the
following drugs should be avoided in this patient during the postoperative period?
A) Albuterol
B) Diphenhydramine
C) Neostigmine
D) Propranolol
E) Varenicline
The correct response is Option B.
Postoperative delirium is an acute brain dysfunction that is characterized by changes in levels of
consciousness, inattention, and disorganized thinking. There are two types of delirium. Delirium can
manifest with hyperactive signs (agitation, restlessness), or hypoactive signs (lethargy,
inattentiveness). It is very common in hospitalized patients, with 60 to 80% of mechanically ventilated
patients and 20 to 50% of patients with a lower severity of illness developing delirium at some point
during their hospitalization.
For patients at risk of postoperative delirium, benzodiazepines and antihistamines should be avoided, as
these medications could exacerbate the symptoms.
Reference(s)
1. Card E, Pandharipande P, Tomes C, et al. Emergence from general anaesthesia and evolution of delirium signs in the post-anesthesia care unit. Br J Anaesth. 115(3):411-7, September 2015.
2. Chow WB, Rosenthal RA, Merkow RP, et al. Optimal preoperative assessment of the geriatric surgical
patient: a best practices guideline from the American College of Surgeons National Surgical Quality
Improvement Program and the American Geriatrics Society. J Am Coll Surg. 215(4):453-66, October
2012.
3. Moskowitz EE, Overbey DM, Jones TS, et al. Post-operative delirium is associated with increased 5-
year mortality. Am J Surg. 214(6):1036-1038, December 2017.
A 52-year-old woman comes to the office after undergoing uncomplicated rhytidectomy 1 week ago. The
patient reports that she cannot feel her left earlobe. Damage to a sensory nerve is suspected. The
affected nerve was most likely injured intraoperatively in which of the following locations?
A) Along a line from the external auditory canal to the lateral edge of the inferior orbit
B) At the anterior border of the sternocleidomastoid muscle
C) Half the distance from mastoid process to the clavicular origin of the sternocleidomastoid muscle
D) One centimeter caudal to the external jugular vein as it crosses the sternocleidomastoid muscle
E) One-third the distance from external auditory canal to the clavicular origin of the sternocleidomastoid
muscle
The correct response is Option E.
The great auricular nerve (GAN) is the most commonly injured nerve in rhytidectomy. The GAN
complication rate is approximately 6.5%. GAN injury can result in pure anesthesia, partial parasthesia, as
well as painful neuromas in the distribution of the nerve. Permanent complete numbness has been
reported in up to 5% of patients. This may cause difficulty wearing earrings, using the telephone, shaving,
or combing one’s hair. Although not as catastrophic as a facial nerve injury, this complication can present
as a functional impairment and nuisance to the patient and surgeon alike.
It is critical to be aware of this nerve when embarking upon rhytidectomy in order to prevent iatrogenic
injury. Once the GAN emerges onto the anterior surface of the sternocleidomastoid muscle, it resides in a
superficial plane and is vulnerable to injury during elevation of facial flaps. The GAN is found at its most
superficial location approximately one third the distance from the external auditory canal to the clavicular origin of the sternocleidomastoid (SCM). A similar distance ratio exists from the mastoid process to the
clavicular origin of the SCM. It also lies approximately one centimeter cranial to the external jugular vein
on the anterior surface of the sternocleidomastoid muscle. With these anatomical landmarks, the surgeon
can accurately predict the location of the GAN at its most vulnerable site and reliably proceed with flap
dissection in the lateral neck during rhytidectomy procedures.
Frankfort’s line is a cephalometric measurement that runs from the external auditory canal to the lateral
edge of the inferior orbit and would be too cranial a location to find the GAN.
Reference(s)
1. Murphy R, Dziegielewski P, O’Connell D, et al. The great auricular nerve: an anatomic and surgical
study. J Otolaryngol Head Neck Surg. 2012 Apr;41 Suppl 1:S75-7.
2. Ozturk CN, Ozturk C, Huettner F, et al. A Failsafe Method to Avoid Injury to the Great Auricular Nerve.
Aesthet Surg J. 2014 Jan 1;34(1):16-21.
3. Rohrich RJ, Taylor NS, Ahmad J, et al. Great auricular nerve injury, the “subauricular band”
phenomenon, and the periauricular adipose compartments. Plast Reconstr Surg. 2011;127(2):835-843.
A 75-year-old woman with type 1 diabetes mellitus undergoes closure of a sternotomy wound using
pectoralis major muscle flaps. On postoperative day 2, her plasma creatinine level has increased to 2.2
from 1.1 mg/dL preoperatively. The patient is hemodynamically stable in the ICU, and her central venous
pressure is within normal range. An intravenous infusion of normal saline is initiated. Which of the
following is the most appropriate next step in management?
A) Administration of a diuretic
B) Discontinuation of enteral nutrition and initiation of parenteral nutrition
C) Discontinuation of protein intake
D) Infusion of low-dose (<2.5 μg/kg/min) dopamine intravenously
E) Plasma glucose control protocol
The correct response is Option E.
This patient has acute kidney injury (AKI) after a surgical procedure. International practice guidelines
recommend insulin therapy for targeted glucose control in critically ill patients. Although the Kidney
Disease – Improving Global Outcomes (KDIGO) task force recommended a plasma glucose target of 110
to 149 mg/dL, the latest recommendation by the Surviving Sepsis Campaign is for an upper blood
glucose level not higher than 180 mg/dL.
Other recommendations for prevention and treatment of AKI by the 2012 KDIGO Clinical Practice
Guideline included:
Isotonic crystalloids rather than colloids (albumin or starches) as initial management for expansion of
intravascular volume in patients at risk for or with AKI; Avoiding restriction of protein intake with the aim
of preventing or delaying initiation of renal replacement therapy (RRT); Administration of 0.8 to 1.0
g/kg/d of protein in non-catabolic AKI patients without need for dialysis; 1.0 to 1.5 g/kg/d in patients with
AKI on RRT; and up to a maximum of 1.7 g/kg/d in patients on continuous renal replacement therapy
(CRRT) and in hypercatabolic patients; Providing nutrition preferentially via the enteral route in patients
with AKI; Not using diuretics to prevent AKI; Not using diuretics to treat AKI, except in the management
of volume overload; Not using low-dose dopamine to prevent or treat AKI
Reference(s)
1. Kellum JA, Lameire N. Guideline Work Group. Diagnosis, evaluation, and management of acute
kidney injury: a KDIGO summary (Part 1). Crit Care. 2013; 17(1):204.
2. Lameire N, Kellum JA. Guideline Work Group. Contrast-induced acute kidney injury and renal support
for acute kidney injury: a KDIGO summary (Part 2). Crit Care. 2013 4; 17(1):205.
3. Rhodes A, Evans LE, Alhazzani W, et al. Surviving Sepsis Campaign: International Guidelines for
Management of Sepsis and Septic Shock: 2016. Crit Care Med. 2017; 45(3):486-552.
A 79-year-old woman undergoes excision and elective neck dissection of a 2.5-cm invasive squamous
cell carcinoma of the right lateral surface tongue. She has no history of head and neck cancer, and there
is no radiologic or clinical evidence of nodal or metastatic disease. Wide excision with adequate margins
and ipsilateral modified radical neck dissection is performed. Elective neck dissection is most likely to
result in which of the following outcomes in this patient?
A) Decreased local recurrence
B) Fewer postoperative complications
C) Increased incidence of distant metastasis
D) Increased nodal relapse
E) Increased overall survival
The correct response is Option E.
This patient will have increased overall survival compared with a patient who does not have elective neck dissection. The patient described has Stage II (T2 N0 M0) oral cancer (larger than 2 cm but not larger
than 4 cm, has not spread to lymph nodes with no metastatic disease). There has been much debate
regarding management of the neck in patients with early-stage oral cancers. The two primary options
include elective neck dissection (ie, at the time of the primary tumor resection) versus therapeutic neck
dissection in the case of nodal relapse. In a prospective, randomized, controlled trial study of patients
with T1 or T2 node-negative oral squamous cell carcinoma, patients received either elective neck
dissection at the time of primary tumor resection or therapeutic neck dissection after nodal relapse. At
3 years, patients who underwent elective neck dissection had a higher rate of survival compared with the
therapeutic surgery group (69.5 vs. 45.9%, P<0.001).
Patients who undergo elective neck dissection at the time of primary tumor resection have an increased
number of postoperative complications and decreased nodal recurrence. Distant metastasis was the
same between the two groups.
Reference(s)
1. D’Druz AK, Vaish R, Kapre N, et al. Elective versus Therapeutic Neck Dissection in Node-Negative
Oral Cancer. N Engl J Med. 2015;337(6):521-529.
2. Thiagarajan S, Nair S, Nair D, et al. Predictors of prognosis for squamous cell carcinoma of oral
tongue. J Surg Oncol. 2014 Jun;109:639-644.
A 43-year-old woman who is BRCA-positive is scheduled to undergo bilateral mastectomy. Tissue
expander–based reconstruction is planned. Which of the following is the optimal duration of antibiotic
prophylaxis for this patient?
A) No preoperative antibiotic
B) One preoperative antibiotic dose and another dose duringskin closure
C) One preoperative antibiotic dose, followed by 24 hours of treatment while in the hospital
D) One preoperative antibiotic dose, followed by 24 hours of treatment while in the hospital and then discharge on oral
antibiotics until drains are removed
E) One preoperative antibiotic dose, followed by 24 hours of treatment while in the hospital and then maintenance on
oral antibiotics until tissue expanders are exchanged
The correct response is Option C.
The overall complication rate in breast reconstructive surgery is as high as 60%. Infection rates can
exceed 20%, much higher than in clean elective surgery. The CDC guidelines suggest only 24 hours of
peri-operative antibiotics beginning thirty minutes prior to skin incision. However, not all plastic surgeons
agree with this. A 2013 meta-analysis found when comparing combined patient cohorts receiving no
antibiotics, antibiotics for less than 24 hours, and antibiotics for greater than 24 hours, the average
infection rates were 14.4, 5.8, and 5.8%, respectively. This demonstrated that the administration of
antibiotics made a difference, however duration beyond 24 hours did not.
A study was published in 2013 evaluating the difference in surgical site infection between two different
prophylactic antibiotic durations (24 hours and until drain removal). In this prospective, randomized,
controlled non-inferiority trial, 24 hours of antibiotics is equivalent to extended oral antibiotics for surgicalsite
infection in tissue expander immediate breast reconstruction patients.
Reference(s)
1. Phillips BT, Bishawi M, Dagum AB, et al. A systematic review of antibiotic use and infection in breast
reconstruction: what is the evidence? Plast Reconstr Surg. 2013;131:1-13.
2. Phillips BT, Bishawi M, Dagum AB, et al. A systematic review of infection rates and associated
antibiotic duration in acellular dermal matrix breast reconstruction. Eplasty. 2014;14:e42.
3. Phillips BT, Fourman MS, Bishawi M, et al. Are Prophylactic Postoperative Antibiotics Necessary for
Immediate Breast Reconstruction? Results of a Prospective Randomized Clinical Trial. J Am Coll Surg.
2016 Jun;222(6):1116-24.
A 67-year-old man undergoes ventral hernia repair and abdominal wall reconstruction with component
separation. On postoperative day 5, the patient develops a cough; temperature is 39.0°C (102.2°F).
Chest x-ray study shows right middle lobe pneumonia. Antibiotic therapy is promptly initiated. Despite
adequate fluid resuscitation, the patient becomes hypotensive (mean arterial pressure < 65 mmHg).
Which of the following blood tests is most appropriate to establish the suspected diagnosis of septic
shock?
A) Albumin
B) C-reactive protein
C) Lactate
D) Plasminogen
E) White blood cell count
The correct response is Option C.
Obtaining a serum lactate level is the most appropriate next step for the diagnosis of septic shock in this
scenario. Patients with septic shock can be clinically identified by having both of two criteria:
1) Vasopressor requirement to maintain a mean arterial pressure of 65 mmHg or greater and
2) Serum lactate level greater than 2 mmol/L (>18 mg/dL) in the absence of hypovolemia.
In 1991, a consensus task force developed initial definitions that focused on the prevailing view at the
time that sepsis resulted from a host’s systemic inflammatory response syndrome (SIRS) to infection.
SIRS was defined by the presence of two or more of four criteria, including body temperature, heart rate,
respiratory rate, and white blood cell count. Despite their known limitations, these definitions remained
mainly unchanged for almost three decades. In 2016, the Society of Critical Care Medicine and the
European Society of Intensive Care Medicine sponsored a task force to review the definition of sepsis
and its management guidelines (Sepsis-3).
Sepsis is now defined as a life-threatening organ dysfunction caused by a dysregulated host response to
infection. This organ dysfunction can be represented by an increase in the Sequential [Sepsis-related]
Organ Failure Assessment (SOFA) score of two points or more. Another measure called quick SOFA (qSOFA), although less robust, may be more practical for providers diagnosing sepsis in the non-ICU
setting. qSOFA incorporates altered mentation (GCS <15), systolic blood pressure of 100 mmHg or less,
and respiratory rate of 22/min or greater.
Septic shock is a subset of sepsis with profound circulatory and cellular/metabolic dysfunction,
associated with a higher risk of hospital mortality than with sepsis alone (40% versus 10%, respectively).
The term “severe sepsis,” previously defined as sepsis complicated by organ dysfunction, has been
incorporated into the current definition of sepsis and abandoned.
Reference(s)
1. Singer M, Deutschman CS, Seymour CW, et al. The Third International Consensus Definitions for
Sepsis and Septic Shock (Sepsis-3). JAMA. 2016; 315(8): 801-810.
2. Rhodes A, Evans LE, Alhazzani W, et al. Surviving Sepsis Campaign: International Guidelines for
Management of Sepsis and Septic Shock: 2016. Crit Care Med. 2017; 45(3): 486-552.
3. Shankar-Hari M, Phillips GS, Levy ML, et al. Developing a New Definition and Assessing New Clinical
Criteria for Septic Shock: For the Third International Consensus Definitions for Sepsis and Septic Shock
(Sepsis-3). JAMA. 2016. 23; 315(8): 775-787.
Which of the following patients is eligible to be an organ donor?
A) 30-year-old HIV-positive patient
B) 10-year-old whose family does not want to donate
C) 42-year-old with a diagnosis of Creutzfeldt-Jakob disease
D) 49-year-old with a recent diagnosis of stage IV melanoma
The correct response is Option A.
Minors are neither eligible for nor able to receive organ donation without the consent of a parent or
guardian.
Creutzfeldt-Jakob disease or any other prion disease is a contraindication for organ donation of any kind.
Metastatic cancer is a contraindication for organ donation.
HIV status is no longer a contraindication to donate or receive an organ, provided the donor and recipient
are both HIV positive.
Reference(s)
1. U.S. Government Information on Organ Donation and Transplantation. Www.Organdonor.gov.
Accessed May 25, 2018.
2. Malani PN. New law allows organ transplants from deceased HIV-infected donors to HIV-infected
recipients. JAMA. 2013; 310:2492.
Which of the following mechanisms is most likely to inhibit normal wound healing in a patient who
smokes cigarettes?
A) DNA strand breaks and helical cross-linking
B) Increased cosubstrate for enzymes involved in collagen production
C) Increased platelet aggregation
D) Increased tissue oxygen delivery
E) Nicotine-induced vasodilation
The correct response is Option C.
The detrimental effects of smoking on wound healing are due primarily to nicotine, carbon monoxide, and
hydrogen cyanide. One of the effects of nicotine is increased platelet aggregation due to enhanced
adhesiveness of the platelets themselves. This leads to thrombus formation and decreased oxygen
delivery. Nicotine does not produce vasodilation, but rather vasoconstriction. Both of these effects
can lead to local tissue ischemia, which inhibits the normal wound healing process.
One of the major mechanisms by which ionizing radiation inhibits wound healing is production of DNA
strand breaks and helical cross-linking, but smoking is not significantly involved.
Vitamin C is the vitamin which plays the greatest role in wound healing. It is required as a cosubstrate for
hydroxylase enzymes, which are involved in the production of collagen. Vitamin C deficiency has long
been known to inhibit wound healing (scurvy). However, supplemental vitamin C in the
nondeficient patient has not been shown conclusively to produce any beneficial wound-healing effects.
Reference(s)
1. Janis JE, Harrison B. Wound Healing: Part I. Basic Science. Plast Reconstr Surg. 2016 Sep; 138(3
Suppl):9S-17S.
2. Silverstein P. Smoking and wound healing. Am J Med. 1992;93(1A):22S-24S.
An otherwise healthy 52-year-old woman comes to the office forconsultation for bilateral mastopexy. Her
last mammogram 2 years ago was negative. Physicalexamination shows a palpable breast mass in the
upper outer quadrant of the right breast that the patient has not noticed previously. Which of the following
is the most appropriate next step in management?
A) Core needle biopsy
B) Diagnostic mammogram with ultrasound
C) Fine-needle aspiration biopsy
D) Mastopexy with open biopsy
E) Screening mammogram
The correct response is Option B.
The first step in the management of a newly found palpable breast mass is x-ray imaging to further
characterize the tumor. The type of imaging required typically depends on the age of the patient at
presentation. In females less than 30 years of age, ultrasound is typically the first (and possibly only) test
ordered as the breast tissue is typically denser and mammography is not as effective. In women greater
than 30 years of age, mammogram is usually the first test ordered. Mammography can evaluate both
breasts for other incidental findings as well as further characterize the mass. Unless the results of the initial mammogram are definitive of a benign etiology of the mass, then an ultrasound is typically
necessary as well. Ultrasound can distinguish cystic from solid masses and will help delineate the
shape, borders, and acoustic properties of the mass. When the mass is suspicious, biopsy is guided by
ultrasonography but this is typically not the initial treatment.
Mammography can be used for both screening and diagnosis. Screening mammography consists of two
routine views, craniocaudal and mediolateral oblique, and is appropriate for asymptomatic
patients. Diagnostic mammography incorporates additional views (e.g. tangential or spot compression
views) in order to better delineate the area of concern. The current patient has a new finding of palpable
mass on exam and requires a diagnostic mammogram for proper evaluation and management.
Given the patient’s age and presentation with newly palpable mass, x-ray imaging prior to any surgery is
warranted to rule out malignancy. Proceeding with surgery that would rearrange the breast tissue may
compromise the oncologic management of a possible breast cancer with incomplete excision and inability
to obtain reliable margins that would require a completion mastectomy instead of the option for breastconserving
therapy.
Reference(s)
1. Brown AL, Phillips J, Slanetz PJ, et al. Clinical Value of Mammography in the Evaluation of Palpable
Breast Lumps in Women 30 Years Old and Older. AJR Am J Roentgenol. 2017 Oct;209(4):935-942.
2. Harvey JA, Mahoney MC, Newell MS, et al. ACR Appropriateness Criteria Palpable Breast Masses. J
Am Coll Radiol. 2013 Oct;10(10):742-9.e1-3.
3. Stein L, Chellman-Jeffers M. The Radiologic Workup of a Palpable Breast Mass. Cleve Clin J Med.
2009; 76(3): 175-180.
A newborn male is brought to the tertiary multidisciplinary referral center for evaluation of anorectal
malformation, tracheoesophageal fistula and absent right thumb. Which of the following associated
VACTERL diagnoses is the best predictor of inpatient mortality?
A) Aniridia with brain stem hypoplasia
B) Anomalies of spine or vertebrae
C) Cardiac disease
D) Renal or urinary anomaly
E) Tracheal stenosis with stridor
The correct response is Option C.
Anomalies of the spine or vertebrae (V), anorectal malformations (A), congenital cardiac anomalies (C),
esophageal atresia/tracheoesophageal fistula (TE), renal and urinary abnormalities (R), and limb lesions
(L) frequently co-occur and are recognized as VACTERL anomalies. VACTERL association is typically
diagnosed in the presence of at least three characteristic features in the absence of evidence for an
overlapping condition, and is estimated to occur in approximately 1 in every 10,000 to 40,000 live births.
The presence of either anorectal malformation or esophageal atresia alone generally triggers a workup
for associated VACTERL diagnoses because of their significant impact on morbidity and mortality. For
example, in a large cohort of children undergoing surgical repair of anorectal malformations, Lautz et al.
found associated VACTERL diagnoses including congenital heart disease in 40.4%, renal or internal
urinary disease in 34.7%, spinal or vertebral anomalies in 31.4%, esophageal atresia/tracheoesophageal
fistula in 7%, and limb defects in 5.6%. The most common limb defects in VACTERL association include
poorly developed or missing thumbs, or underdeveloped forearms and hands, polydactyly, syndactyly,
and reduction deformities of the lower limb.
Independent predictors of mortality in any patient with VACTERL association include congenital heart
disease (greatest for those who require cardiac surgery than those with a diagnosis but no operation),
birthweight <2 kg, and black race. Of note, the association between cardiac disease and higher mortality
has been reproduced in several studies.
Aniridia, brain stem hypoplasia, and tracheal stenosis with stridor are not primary characteristics of
VACTERL association.
Reference(s)
1. Lautz TB, Mandelia A, Radhakrishnan J. VACTERL associations in children undergoing surgery for
esophageal atresia and anorectal malformations: Implications for pediatric surgeons. J Pediatr Surg.
2015;50(8):1245-1250.
2. Olgun H, Karacan M, Caner I, Oral A, Ceviz N. Congenital cardiac malformations in neonates with
apparently isolated gastrointestinal malformations. Pediatr Int. 2009;51(2):260-262.
Totonelli G, Catania VD, Morini F, et al. VACTERL association in anorectal malformation: effect on the outcome. Pediatr Surg Int. 2015;31(9):805-808.
A 33-year-old transfeminine (male-to-female) patient with gender dysphoria presents for consultation
regarding bilateral breast enlargement with silicone implants. Which of the following is the most
appropriate CPT code for this procedure?
A) 19324-50: mammaplasty, augmentation; without prosthetic implant
B) 19325-50: mammaplasty, augmentation; with prosthetic implant
C) 19342-50: delayed insertion of prosthesis in breast reconstruction
D) 19357-50: immediate insertion of a tissue expander
E) 19366-50: breast reconstruction with other technique
The correct response is Option B.
Breast surgery for treatment of gender dysphoria is a recognized therapeutic option, which is covered by
the Centers for Medicaid and Medicare Services, military health maintenance organizations, and most
private payers. Because breast reconstruction with implants is a defined, covered benefit for women with
breast cancer, as mandated by federal legislation, the US judicial system has ruled that this procedure
should also be available to transgender women who desire breast reconstruction. Because this benefit is
available for some women, this benefit should be available for all women, including transgender women.
Withholding a medically necessary procedure for treatment of gender dysphoria would represent a form
of gender discrimination.
The CPT code recognized by both private and public health insurance companies is 19325-50 for
bilateral augmentation mammoplasty with prosthetic implant. Even though this code is most often used in
the aesthetic setting, the procedure is considered to be reconstructive in transgender women with gender
dysphoria.
Reference(s)
1. CPT corner: Coding for sex-reassignment surgery is evolving. Plastic Surgery News, March 2015,
page 14.
2. CMS.gov. Gender Reassignment Surgery Model NCD. Page 6.
https://www.cms.gov/medicare/coverage/determinationProcess/downloads/Kalra_comment_01022016.pdf.
A 51-year-old farmer is brought to the emergency department after sustaining extensive burns in a
fertilizer explosion. Examination shows white phosphorus embedded in his burn wounds. In addition to
burn resuscitation and examination of the wounds under ultraviolet light, application of which of the
following is the most appropriate next step in management?
A) Calcium gluconate
B) Mafenide (Sulfamylon)
C) Mineral oil
D) Polyethylene glycol
E) Saline irrigation
The correct response is Option E.
White phosphorus is sustained in both military and civilian circumstances. It is commonly found in
fireworks, fertilizers, and pesticide. It is extremely volatile and can ignite spontaneously upon exposure to
air. Additionally, phosphoric acids form during combustion and further injure tissues.
Treatment mainstays include:
1. Immediate debridement of visible debris
2. Copious irrigation
3. Keep the area wet and covered with saline-soaked gauze
4. Cardiac monitoring and electrolyte evaluation.
Profound hypocalcemia, hyperphosphatemia, and sudden death have been associated with this injury.
Calcium gluconate gel is used in the management of hydrofluoric acid burns. Polyethylene glycol is used
in the management of phenol and cresol burns. Mineral oil is used to isolate potassium, sodium, and
magnesium from water, with which they react explosively. Mafenide (Sulfamylon) has no role in the immediate management of white phosphorus burns.
Reference(s)
1. Renz E, Cancio L. Chapter 12, Acute Burn Care. In: Combat Casualty Care. Falls Church, VA: Office
of the Surgeon General of the United States Army; 2012: 596-631.
2. Hardwicke J, Hunter T, Staruch R, Moiemen N. Chemical burns-an historic comparison and review of
the literature. Burns. 2012 May;38(3):383-7.
3. Barillo DJ, Cancio LC, Goodwin CW. Treatment of white phosphorus and other chemical burn injuries
at one burn center for over a 51 year period. Burns. 2004 Aug;30(5):448-52.
In a randomized, blinded, placebo controlled trial, 84% of patients in the migraine surgery vs. 58% of
patients in the sham surgery group had greater than 50% reduction in migraine symptoms (p < 0.05).
Which of the following is indicated by a p value of <0.05?
A) The observed difference is likely due to sampling variation (accept null hypothesis)
B) The observed difference is likely due to sampling variation (reject null hypothesis)
C) The observed difference is not likely due to sampling variation (accept null hypothesis)
D) The observed difference is not likely due to sampling variation (reject null hypothesis)
E) Cannot make a determination regarding the null hypothesis
The correct response is Option D.
The p value is defined as the probability of getting a difference at least as large as that observed if the
null hypothesis is true. The larger the p value, the more likely the observed difference is due to sampling
error (and therefore one accepts the null hypothesis of no difference). The smaller the p value, the more
likely the observed difference is not due to sampling error (and therefore one rejects the null hypothesis
of no difference).
In patients who suffer from moderate to severe migraine headaches from a single or predominant trigger
site, 84% of patients that underwent surgical decompression of that trigger point experienced reduction in migraine symptoms by more than 50%, compared to 58% of those who underwent sham surgery.
Reference(s)
1. Guyuron B, Reed D, Kriegler JS, et. al. A Placebo-Controlled Surgical Trial of the Treatment of
Migraine Headaches. Plast Reconstr Surg. 124:461, 2009.
2. Ruffenburgh RH. Statistics in Medicine. 3 rd edition. San Diego, California; Elsevier Inc; 2012.
Which of the following forms of communication is compliant with the Health Insurance Portability and
Accountability Act (HIPAA)?
A) E-mailing the confidential information using an encrypted patient portal server
B) Leaving protected information on the patient’s voicemail
C) Placing a sealed folder with patient records under the attending physician’s office door
D) Texting medical information to a password protected smart phone
E) Transferring the patient records via a non-encrypted flash drive
The correct response is Option A.
The Health Insurance Portability and Accountability Act of 1996 (HIPAA) is United States legislation that
provides data privacy and security provisions for safeguarding medical information. Unauthorized release
of any confidential or identifying information, which can be linked to an individual patient, is considered a
violation of the law, with penalties ranging from fines to incarceration. The secure, private transmission of
Protected Health Information (PHI) is allowed between two treating health-care professionals, provided
that the communication is confidential and not at significant risk of breach or theft. Transmission of PHI
via social media, e-mail, and other electronic methods must be done through a combination of
safeguards that involves encryption. Although the legal understanding of how to communicate PHI
continues to evolve, these devices must meet institutional requirements for security.
Reference(s)
1. Lifchez SD, McKee DM, Raven RB 3rd, et al. Guidelines for ethical and professional use of social media in a hand surgery practice. J Hand Surg Am. 2012 Dec;37(12):2636-41.
2. Drolet BC, Marwaha JS, Hyatt B, et al. Electronic Communication of Protected Health Information:
Privacy, Security, and HIPAA Compliance. J Hand Surg Am. 2017 Jun;42(6):411-416.
3. Gordon CR, Rezzadeh KS, Li A, et al. Digital mobile technology facilitates HIPAA-sensitive
perioperative messaging, improves physician-patient communication, and streamlines patient care.
Patient Saf Surg. 2015 May 23;9:21. Healthcare reimbursement: 13.I.1
An 81-year-old man with peripheral vascular disease undergoes coverage of exposed vascular
prosthesis in the groin with a rectus femoris muscle flap. On postoperative day 2, the patient has sudden
onset of chest tightness and becomes unresponsive, with no palpable pulse. CPR is promptly initiated.
The defibrillator monitor shows ventricular fibrillation. Which of the following is the most appropriate next
step after shock delivery (electrical defibrillation)?
A) Capnometry
B) Chest compressions for 2 minutes
C) Endotracheal intubation
D) Intravenous administration of adenosine
E) Pulse/rhythm check
The correct response is Option B.
According to current Advanced Cardiac Life Support (ACLS) guidelines, CPR should be resumed
immediately after shock delivery, without pausing for a rhythm or pulse check. It should begin with chest
compressions and continue for 2 minutes, after which the rhythm should be checked and the cycle
(shock/CPR 2 min/check) repeated if there is no return of spontaneous circulation (ROSC).
Increasing emphasis has been placed on the importance of continuous “high-quality” chest compression
(5 cm sternal depression, 100 to 120/min), to maximize tissue perfusion and probability of ROSC. A 30:2
compression:ventilation rate is recommended in the absence of an endotracheal or supraglottic airway.
Otherwise, 10 breaths per minute should be delivered with continuous chest compressions.
There are no studies directly addressing the timing of advanced airway placement and outcome during
resuscitation from cardiac arrest. Although insertion of an endotracheal tube during ongoing chest
compressions is possible, in most instances intubation is associated with interruption of compressions for
many seconds. Particularly, patients with witnessed cardiac arrest from ventricular fibrillation or pulseless
ventricular tachycardia may benefit from a few uninterrupted cycles of CPR prior to placement of an
advanced airway.
Adenosine is not indicated in the treatment of adult cardiac arrest.
Capnometry/capnography requires placement of an endotracheal tube.
Reference(s)
1. Link MS, Berkow LC, Kudenchuk PJ, et al. Part 7: Adult Advanced Cardiovascular Life Support: 2015
American Heart Association Guidelines Update for Cardiopulmonary Resuscitation and Emergency
Cardiovascular Care. Circulation. 2015 Nov 3;132(18 Suppl 2):S444-64.
2. Bobrow BJ, Ewy GA, Clark L, et al. Passive oxygen insufflation is superior to bag-valve-mask
ventilation for witnessed ventricular fibrillation out-of-hospital cardiac arrest. Ann Emerg Med. 2009; 54:
656-662.
3. Zhan L, Yang LJ, Huang Y, et al. Continuous chest compression versus interrupted chest compression
for cardiopulmonary resuscitation of non-asphyxial out-of-hospital cardiac arrest. Cochrane Database
Syst Rev. 2017;3:CD010134.
A 64-year old woman comes to the office because of a non healing radiated scalp wound. Medical history
includes resection of invasive basal cell carcinoma of the scalp, reconstruction with a scalp rotation flap,
and high-dose postoperative radiation therapy (60 Gy) 10 years ago. Clinical examination shows a fullthickness
wound consisting of erythematous, ulcerated, and necrotic skin, and exposed, foul-smelling
skull at the base of the wound. Which of the following is the most appropriate next step in management of
this patient?
A) Biopsy of the wound
B) Craniectomy with free flap reconstruction
C) MRI
D) Resection of involved scalp with split-thickness skin grafting
E) Vacuum-assisted closure (VAC)
The correct response is Option A.
The first step in managing this patient is biopsy of the wound to rule out cancer recurrence. Although the
diagnosis is most likely osteoradionecrosis of the skull, one would not proceed with the next steps of
management until recurrence of cancer is ruled out. In this patient, the management sequence would
include a biopsy to rule out cancer recurrence, followed by CT scan to delineate the extent of the skull
involvement. MRI would not delineate the extent of the bony involvement.
The rates of osteoradionecrosis occurrence vary in the literature (from 1.8 to 37%). Although the rate and
severity of osteoradionecrosis are most consistently associated with doses of radiation exceeding 50 Gy,
there are reports of osteoradionecrosis in patients who received doses as low as 30 Gy.
This patient would require extensive craniectomy by a neurosurgeon to debride the wound of necrotic
bone and, in most cases, reconstruction with free tissue transfer. Vacuum-assisted closure would not be
a viable option for this patient, nor would resection of the scalp with split-thickness skin grafting.
Reference(s)
1. Nguyen MT, Billington A, Habal MB. Osteoradionecrosis of the Skull After Radiation Therapy for
Invasive Carcinoma. J Craniofac Surg. 2011;22(5):1677-1681.
2. Huang WB, Wong STS, Chan JYW. Role of surgery in the treatment of osteoradionecrosis and its
complications after radiotherapy for nasopharyngeal carcinoma. Head Neck. 2017 Oct 9.
A 12-year-old boy is brought to the office for evaluation of an obvious anterior open bite sustained when
he fell from his bicycle. Imaging shows a displaced fracture of the right mandibular condyle with intraarticular
extension. Which of the following is the most appropriate treatment?
A) Application of an external fixator
B) Delayed sagittal split osteotomy
C) Maxillomandibular fixation with arch bars
D) Open reduction and internal fixation
E) Soft diet and observation
The correct response is Option C.
Condylar fractures in children can predispose to facial growth disturbance and temporomandibular joint
dysfunction. A condylar fracture with an associated parasymphyseal fracture and an open bite should be
treated with arch bars and a period of intermaxillary fixation in a 12-year-old, if possible. Open reduction
and plating of the mandible is generally avoided in this age group to avoid injury to tooth buds. It has
been demonstrated that arch bars can be used safely and effectively for the injury pattern described
during the period of mixed dentition. Delayed sagital split osteotomy is not indicated in a patient with a
normal premorbid occlusion.
Reference(s)
1. Ghasemzadeh A, Mundinger GS, Swanson EW, et al. Treatment of Pediatric Condylar Fractures: A
20-Year Experience. Plast Reconstr Surg. 2015 Dec;136(6):1279-88.
2. Naran S, Keating J, Natali M, et al. The safe and efficacious use of arch bars in patients during primary
and mixed dentition: a challenge to conventional teaching. Plast Reconstr Surg. 2014 Feb;133(2):364-6.
A 69-year-old man is brought to the emergency department because of acute onset of excruciating pain
of the left forearm and a pale, pulseless, cool left hand. Medical history includes atrial fibrillation and
steroid-dependent chronic obstructive pulmonary disease (COPD). Physical examination and hand-held
Doppler interrogation are consistent with acute arterial blockage in the left ulnar artery. In addition to
aspirin, which of the following therapeutic interventions should be administered promptly while assessing
the patient’s ability to withstand surgery?
A) Apixaban
B) Clopidogrel
C) Heparin
D) Recombinant tPA
E) Verapamil
The correct response is Option C.
Iannuzzi and colleagues have summarized the treatment for acute arterial thrombosis of the hand.
Prevention of further damage should be the first line of treatment while completing work-up of the patient.
They recommend that heparin and aspirin should be administered to prevent propagation of further
arterial occlusion. While the idea of reducing vasospasm in the surrounding vessels is appealing,
Iannuzzi’s review of the literature is inconclusive of any benefit for tissue salvage outcomes. The article is
also useful for comparison of the various imaging modalities for definitive diagnosis and approach to
treatment.
In their meta-analysis for the Cochrane library, Berridge et al. surveyed the literature and came to the
conclusion that distal limb salvage was similar at 30 days, 6 months, and 1 year with either surgical
extraction of clot or thrombolysis by direct delivery of the agent to the artery in question. Bleeding and
distal embolization were more common after use of thrombolytic agents at 30 days.
Robertson et al, also in meta-analysis for the Cochrane library, found some differences favoring tissue
plasminogen activator (tPA) in initial vessel patency, but there were no differences in limb salvage
outcomes with intra-arterial delivery of tPA or urokinase. In the streptokinase vs tPA studies, there were
increased bleeding complications noted with streptokinase.
Reference(s)
1. Robertson I, Kessel DO, Berridge DC. Fibrinolytic agents for peripheral arterial occlusion. Cochrane
Database Syst Rev. 2010 Mar 17;(3):CD001099.
2. Iannuzzi NP, Higgins JP. Acute Arterial Thrombosis of the Hand. J Hand Surg Am. 2015
Oct;40(10):2099-106.
3. Berridge DC, Kessel DO, Robertson I. Surgery versus thrombolysis for initial management of acute
limb ischaemia. Cochrane Database Syst Rev. 2013 Jun 6;(6):CD002784.
A 59-year-old woman presents with an infected sternal nonunion after coronary artery bypass grafting 4
weeks ago.After debridement of the wound, five sternal plates and bilateral pectoralis flaps are placed.
Postoperatively, the patient becomes hypotensive, tachycardic, and confused. Jugular distention is noted. Oxygen saturation is 100% on nasal cannula. Which of the following is the most appropriate initial
step in management?
A) Auscultation
B) Chest x-ray
C) ECG
D) Ultrasonography of the heart
E) Return the patient to the operating room
The correct response is Option A.
On auscultation a muffled heart sound and pericardial friction rub is heard and would direct the clinician
to decompress tamponade.
Patient is demonstrating Beck’s triad and has reason for possible cardiac tamponade.
Immediate chest x-ray can be ordered to help rule out pneumothorax, but with normal oxygenation, the
chance of a pneumothorax is lower on the differential, and there are other better initial diagnostic and
therapeutic steps.
ECG can help support the diagnosis of pericardial effusion, but this is not diagnostic and is only used as
an adjunct.
Ultrasonography of the heart can confirm the existence of pericardial effusion, as well as allow needle
drainage for immediate treatment. However, this would be performed after auscultation.
Reference(s)
1. Spodick DH. Acute cardiac tamponade. N Engl J Med. 2003; 349:684.
2. Adler Y, Charron P, Imazio M, et al. 2015 ESC Guidelines for the diagnosis and management of
pericardial diseases: The Task Force for the Diagnosis and Management of Pericardial Diseases of the
European Society of Cardiology (ESC)Endorsed by: The European Association for Cardio-Thoracic
Surgery (EACTS). Eur Heart J. 2015; 36:2921.
Compared with standard wound dressings, postoperative negative pressure wound therapy is most likely
to produce which of the following outcomes?
A) Better delayed primary fascial closure rates for salvage laparotomy
B) Better split-thickness skin graft incorporation
C) Increased inflammatory response
D) Increased postoperative dressing changes
E) Increased risk of infection
The correct response is Option B.
Multiple studies have shown the benefits of negative pressure wound therapy (NPWT) when used in
conjunction with skin grafts, both as a bolster over a skin graft as well as wound bed preparation.
NPWT has been shown to decrease the risk of infection in complex and traumatic wounds in some
studies, while others have shown no difference in infection rates in complex wounds when the patient has
multiple comorbidities or when used to cover uncomplicated incisions for elective orthopedic
operations. However, there is no evidence to suggest NPWT increases infection risk compared with
standard wound dressings.
NPWT reduces both inflammatory response and edema formation.
When used for damage control laparotomy and abdominal compartment syndrome, studies have failed to
show any benefit of NPWT over standard dressings. Furthermore, at least one study has suggested an
increased rate of enteric fistula formation is associated with NPWT.
NPWT has been shown to decrease both the number of postoperative dressing changes and the number
of additional operative interventions in complicated diabetic wounds.
Reference(s)
1. Anghel EL, Kim PJ. Negative-pressure wound therapy: a comprehensive review of the evidence. Plast Reconstr Surg. 2016 Sep; 138(3 Suppl): 129S-37S.
2. Evangelista MS, Kim EK, Evans GRD, Wirth GA. Management of skin grafts using negative pressure
therapy: the effect of varied pressure on skin graft incorporation. Wounds. 2013;25(4):89-93.
A 54-year-old woman with a history of augmentation mammaplasty with textured silicone implants has
histologic confirmation of breast implant–associated anaplastic large cell lymphoma (BIA-ALCL). MRI and
PET scans show no associated masses, with activity localized to the periprosthetic seroma. Which of the
following is the most appropriate next step in management of this patient?
A) Anterior capsulectomy with removal of the implants bilaterally
B) Complete capsulectomy with removal of the implant on the affected side
C) Partial capsulectomy with replacement of the implant
D) Removal of the textured implant and replacement with a smooth implant
E) Sealing of the seroma cavity with fibrin glue
The correct response is Option B.
Breast implant–associated anaplastic large cell lymphoma (BIA-ALCL) is a rare peripheral T-cell
lymphoma that has been increasingly recognized as a serious, albeit uncommon, complication
associated with the use of textured breast implants. Since the initial case report in 1996, there have been
continually increasing reported cases of this rare malignancy and according to the most recent data
available, the lifetime risk of association between breast implants and BIA-ALCL is between 1 in 1000 to
1 in 30,000 with the ASPS recognizing nearly 200 cases in the US and nearly 500 cases worldwide.
BIA-ALCL patients typically present with a spontaneously occurring periprosthetic fluid collection or
capsule-associated mass approximately 10 years following implantation of the breast implant. To date, all
cases have had some association with a textured device. Initial workup includes ultrasound for evaluation
of a periprosthetic fluid collection or mass. Periprosthetic fluid collections should undergo fine-needle
aspiration in the clinic or ultrasound-guided aspiration by interventional radiology if there is concern for
trauma to the implant while masses require tissue biopsy. Specimens should be sent for cytology with
immunohistochemistry and flow cytometry for T-cell markers, specifically CD30 cell surface protein. A
recent systematic review revealed that 66% of BIA-ALCL patients presented with isolated late-onset seroma while only 8% presented with an isolated new breast mass.
National Comprehensive Cancer Network (NCCN) guidelines for treatment of BIA-ALCL recommend
complete removal of the lymphoma (fluid and/or mass), complete capsulectomy, and removal of the
implant. More advanced disease may require chemotherapy, radiotherapy, and/or lymph node
dissection. Although some surgeons advocate removal of the contralateral breast implant as
approximately 4.6% of cases have demonstrated incidental lymphoma in the contralateral breast, this
recommendation is controversial. The official NCCN guidelines for treatment only recommend
consideration of contralateral breast implant removal but this is not mandated.
Reference(s)
1. Clemens MW, Horwitz SM. NCCN Consensus Guidelines for the Diagnosis and Management of
Breast Implant-Associated Anaplastic Large Cell Lymphoma. Aesthet Surg J. 2017 Mar 1;37(3):285-289.
2. Leberfinger AN, Behar BJ, Williams NC, et al. Breast Implant-Associated Anaplastic Large Cell
Lymphoma: A Systematic Review. JAMA Surg. 2017;152(12): 1161-1168.
A 12-year-old boy is referred to a multidisciplinary sarcoma treatment center because of a deep localized
rhabdomyosarcoma of the right thigh. After neoadjuvant radiotherapy, radical resection with curative
intent, including a 20-cm segmental intercalary resection of involved distal femoral diaphysis, is
performed. Skin and major neurovascular structures will be spared. Postoperative chemotherapy is
planned. Which of the following is the most appropriate method for management of the bony defect in this
patient?
A) Distraction osteogenesis
B) Free fibula transfer with femoral allograft (Capanna technique)
C) Induced membrane (Masquelet) technique
D) Lower extremity rotationplasty (Van Ness procedure)
E) Pedicled medial femoral condyle flap
The correct response is Option B.
Rhabdomyosarcomas represent the most common soft-tissue sarcoma of childhood and are responsible
for approximately half of all soft-tissue sarcomas in this age group. They are thought to originate from
immature cells that are destined to form striated skeletal muscle, although they can arise anywhere in the
body. With modern multimodal management, the cure rates for localized disease are generally greater
than 70% overall. The primary goal of local tumor control in extremity rhabdomyosarcoma is limb-sparing
complete resection where possible.
Vascularized bone grafting represents the gold standard for reconstructing segmental bone loss greater
than 6 cm associated with a compromised local soft-tissue environment that occurs with radiotherapy and
chemotherapy. For large weight-bearing intercalary reconstruction, significant literature supports the
combination of a large structural allograft combined with vascularized fibula as described by Capanna in
1980. With this combination, the neoosteogenic properties of the free fibula are supplemented by the
immediate structural support of the bulk allograft and provide a durable single-stage biological
reconstruction.
Distraction osteogenesis is a technique of de novo bone formation that capitalizes on normal bone
healing with gradual, surgically controlled distraction of adjacent osteotomy defects and has the
advantage of simultaneously expanding surrounding soft-tissue envelopes. The technique requires viable
bone in proximity to one another following a latency phase and is useful in limb lengthening and
craniofacial procedures but has limited utility in long segmental tumor reconstruction.
The induced membrane technique proposed by Masquelet is a two-step procedure where a segment of
bone loss is first filled with an acrylic spacer and later replaced by cancellous bone graft in the so-called
self-induced reactive “periosteal” membrane. The technique requires two stages and is less favored in
the setting of planned radiation or chemotherapy where experience has shown that vascularized flaps or
supplemented vascularized allografts are beneficial.
The medial femoral condyle flap has been used for small osteoperiosteal, corticoperiosteal, and
osteocartilaginous flaps based off either the articular descending genicular or superomedial genicular
arteries. It would be insufficient in size for a 20-cm-long bone defect.
The Van Ness rotationplasty is a type of autograft where functional limb below the knee is used to
reconstruct more proximal defects. It can be a useful “spare part” reconstructive option in composite
proximal extremity resections by repurposing a functional ankle joint more proximally in a rotated
configuration for preserved gait advantage at the repurposed knee. A rotationplasty would not be
indicated for intercalary resections sparing joint and metaphysis.
Reference(s)
1. Capanna R, Campanacci DA, Belot N, et al. A new reconstructive technique for intercalary defects of long bones: the association of massive allograft with vascularized fibular autograft. Long-term results and
comparison with alternative techniques. Orthop Clin North Am. 2007;38(1):51-60.
2. Dasgupta R, Rodeberg DA. Update on rhabdomyosarcoma. Semin Pediatr Surg. 2012;21(1):68-78.
3. Pappo AS, Dirksen U. Rhabdomyosarcoma, Ewing Sarcoma, and Other Round Cell Sarcomas. J Clin
Oncol. 2018;36(2):168-179.
A 48-year-old woman undergoes excision of a 3-cm recurrent keloid of the presternal chest. Immediate
reconstruction with a collagen-glycosaminoglycan scaffold dermal regeneration template is performed,
followed by thin (0.008-in) epidermal autografting 21 days later. After it has healed completely, punch
biopsy is performed. The absence of which of the following histologic features is most likely to indicate
regenerated skin in this patient?
A) Capillary loops at the dermal-epidermal junction
B) Elastic fibers
C) Hair follicles
D) Neovascularization
E) Rete ridges
The correct response is Option C.
Regenerated skin is clearly quite different histologically from scar and, in fact, shares many
characteristics with normal physiologic skin. Regenerated skin shows mechanical competence,
vascularization, and heat and cold sensitivity. Furthermore, the dermal-epidermal junction shows
formation of rete ridges and capillary loops. Regenerated skin displays elastic fibers and increased
collagen fiber density in the reticular dermis, and it often exhibits nerve fiber regeneration as
well. Regenerated skin, even when resurfaced with a split-thickness skin graft, however, does not have
the dermal appendages such as hair follicles and sweat glands, that are present throughout normal skin.
Reference(s)
1. Yannas IV, Orgill DP, Burke JF. Template for skin regeneration. Plast Reconstr Surg. 2011 Jan; 127(1Suppl):60S-70S.
2. Moiemen N, et al. Long-term clinical and histological analysis of integra dermal regeneration template.
Plast Reconstr Surg. 2011 Mar; 127(3):1149-54.
An 80-year-old man sustains an extravasation injury to the dorsum of the arm secondary to
administration of a dopamine infusion. Which of the following findings is an indication for a surgical
intervention in this patient?
A) Blanching of the skin
B) Blistering
C) Erythema
D) Induration
E) Persistent pain
The correct response is Option E.
The indications for surgery in an extravasation injury include full-thickness skin necrosis, chronic
ulceration, and persistent pain. Whereas blistering indicates a partial-thickness skin loss, it is alone not
an indication for surgery. Erythema, induration, and poor capillary refill (blanching) are signs of
extravasation injury but are not indications for an operative intervention.
Reference(s)
1. Al-Benna S, O’Boyle C, Holley J. Extravasation injuries in Adults. ISRN Dermatol. 2013 May
8;2013:856541.
2. Scuderi N, Onesti MG. Antitumor agents: Extravasation, management, and surgical treatment. Ann
Plast Surg. 1994 Jan;32(1):39-44.
A 45-year-old woman undergoes abdominoplasty in an ambulatory surgery center. After induction of
general anesthesia, the patient’s end tidal carbon dioxide level increases, her heart rate increases to 160
bpm, and her arms become rigid. Which of the following are the physiologic abnormalities associated with this condition?
A) Hyperkalemia, hyperphosphatemia, metabolic acidosis
B) Hyperkalemia, hyperphosphatemia, metabolic alkalosis
C) Hyperkalemia, hypophosphatemia, metabolic acidosis
D) Hypokalemia, hyperphosphatemia, metabolic acidosis
E) Hypokalemia, hyperphosphatemia, metabolic alkalosis
The correct response is Option A.
Malignant hyperthermia is an inherited myopathy that is autosomal dominant with variable
penetrance. Anesthetic agents that trigger malignant hyperthermia include halothane, enflurane,
isoflurane, desflurane, sevoflurane, and succinylcholine. These agents trigger an earlier calcium release
into the skeletal muscle, resulting in an abnormal buildup of calcium in the myoplasm. This flood of
calcium causes the muscle to remain in a contracted state, producing high levels of lactic acid, carbon
dioxide, phosphate, and heat. The resulting physiologic changes are metabolic acidosis, hypercapnia,
hyperphosphatemia, and fever in a patient experiencing malignant hyperthermia. The treatment of
malignant hyperthermia is discontinuation of volatile agents and succinylcholine, dantrolene, treatment of
hyperkalemia and metabolic acidosis, and transfer to an acute care hospital.
Reference(s)
1. Gurunluoglu R, Swanson JA, Haeck PC. Evidence-based patient safety advisory: malignant
hyperthermia. Plast Reconstr Surg. 124(Suppl.):68S, 2009.
2. In J, Ahn EJ, Lee DK, et al. Incidence of malignant hyperthermia in patients undergoing general
anesthesia. Medicine. 96(49):e9115, December 2017.
Administration of prophylactic antibiotics is most appropriate for which of the following surgical
procedures?
A) Abdominoplasty
B) Blepharoplasty
C) Brachioplasty
D) Mastopexy
E) Rhytidectomy
The correct response is Option D.
The ASPS recently published the first consensus statement/guidelines for antibiotic prophylaxis in plastic
surgery which is based on comprehensive systematic review of the available evidence.
Systemic antibiotic prophylaxis is recommended for clean-contaminated, contaminated, or dirty plastic
surgery of the head and neck, orthognathic/mandibular, septoplasty/rhinoplasty, hand and upper limb,
and skin. Antibiotic prophylaxis is also recommended to reduce surgical-site infection for clean plastic
surgery of the breast. Antibiotic prophylaxis is not recommended to reduce surgical-site infection in clean
surgical cases of the head and neck, orthognathic/mandibular area, hand and upper limb, skin, and
abdominoplasty.
With the exception of cosmetic breast surgery, clean operations have not been shown to benefit from
routine antibiotic prophylaxis. Clean-contaminated and contaminated plastic surgical procedures do
benefit from the use of antibiotic prophylaxis. The duration of antibiotic use should generally be limited to
a single preoperative dose because studies have generally showed no benefit for longer term antibiotic
prophylaxis.
As far as choosing the antibiotic, it should have activity against the most frequently encountered
microorganisms in postoperative surgical-site infections. Cefazolin as a single dose preoperatively is the
most commonly recommended agent and would be considered appropriate in most cases. In the event of
allergy or intolerance, clindamycin or vancomycin may be appropriate alternatives.
Reference(s)
1. Ariyan A, Martin J, Lal A, et al. Antibiotic Prophylaxis for Preventing Surgical-Site Infection in Plastic
Surgery: An Evidence-Based Consensus Conference Statement from the American Association of Plastic
Surgeons. Plast Reconstr Surg. 2015 Jun;135(6):1723-39.
A 65-year-old woman remains intubated in the intensive care unit after undergoing a prolonged
operation. Arterial blood gas analysis shows respiratory acidosis. An increase in respiratory minute
ventilationis planned. Minute ventilation is calculated by multiplying the respiratory rate and which of the
following parameters?
A) Inspiratory capacity
B) Residual volume
C) Tidal volume
D) Total lung capacity
E) Vital capacity
The correct response is Option C.
Minute ventilation is calculated by multiplying respiratory rate and tidal volume. Tidal volume is the
amount of air/gas displaced during each quiet breath, using no extra inspiratory (“deep breath”) or
expiratory effort.
Minute ventilation is an important concept in mechanical ventilation because of its inverse relationship
with blood carbon dioxide levels. The caveat of this relationship is that not all inhaled air/gas takes part in
gas exchange, whether because it remains in the conductive airways (ventilator tubing, endotracheal
tube, trachea, etc) or it reaches alveoli that are not adequately perfused. The volume of air/gas that does
not take part in gas exchange is called dead space. Residual volume is the volume of air still remaining in the lungs after the most forcible expiration possible.
Inspiratory capacity is the volume of air that enters the lungs during the most forcible inspiration possible,
starting at rest. Inspiratory reserve volume equals inspiratory capacity minus tidal volume, or the
difference between the deepest breath and a quiet breath.
Expiratory reserve volume is the amount of air that can still be expired after a quiet expiration ends. It
requires contraction of expiratory chest wall muscles, as opposed to quiet expiration, which is passive.
Vital capacity is the total amount of air that can be forcefully expired from the lungs after the most forcible
inspiration possible. It represents the addition of inspiratory reserve, tidal, and expiratory reserve
volumes.
Total lung capacity is the combination of vital capacity and residual volume.
Reference(s)
1. Hall, JE, Guyton AC. Pulmonary Ventilation. In: Hall, JE, ed. Guyton and Hall Textbook of Medical
Physiology, 13 th ed. Philadelphia, PA: Elsevier; 2016. Chapter 38: 497-507.
2. Rodriguez-Roisin R, Ferrer A. Chapter 37: Effect of Mechanical Ventilation on Gas Exchange. In:
Tobin, MJ, ed. Principles and Practice of Mechanical Ventilation, 3 rd ed. New York, NY: McGraw-Hill
Medical; 2013.
A 54-year-old woman with a history of left mastectomy for breast cancer presents for right reduction
mammaplasty for symmetry. In this patient, the incidence of occult breast cancer discovered incidentally
in tissue specimens at the time of reduction mammaplasty is approximately which of the following?
A) 0.4 %
B) 1 %
C) 5 %
D) 15 %
E) 23 %
The correct response is Option C.
There have been multiple studies on the incidence of breast cancer discovered in reduction
mammaplasty specimens. The incidence of occult cancer detected in reduction mammaplasty specimens
is typically very low (0.06 to 5.45%) but varies depending on the patient’s age and history of breast
cancer. One specific study compared women undergoing reduction mammaplasty for symptomatic
macromastia with women undergoing reduction mammaplasty for symmetry after mastectomy with or
without reconstruction. Incidentally discovering occult breast cancer was much higher in women
undergoing symmetry procedures (5.5 vs. 0.4%) versus those undergoing reduction mammaplasty for
symptomatic macromastia. The important distinction in this clinical vignette is that the woman has had a
mastectomy for breast cancer, and highlights several important points including:
1) The importance of a thorough history before reduction mammaplasty 2) Preoperative clinical
examination 3) Screening mammography prior to the reduction mammaplasty 4) Pathologic examination
of reduction mammaplasty specimens
Based on multiple studies, the other percentages listed are either too high or too low.
The treatment of occult cancers discovered during reduction mammaplasty depends on several factors
including family history and evaluation of surgical margins.
Reference(s)
1. Carlson GW. The Management of Breast Cancer Detected by Reduction Mammaplasty. Clin Plastic
Surg. 2016:43:341-347.
2. Tadler M, Vlastos G, Pelte MF et al. Breast lesions in reduction mammoplasty specimens: a
histopathological pattern in 534 patients. Br J Cancer. 2014;110(3):788-91.
A 30-year-old woman undergoes augmentation mammaplastyin an office-based operating room.
Intravenous midazolam and fentanyl are used, and a lidocaine field block is administered. An hour later,
while in the recovery room, the patient experiences disorientation, muscle twitching, and lightheadedness.
Administration of which of the following drugs is the most appropriate next step in
management?
A) Dantrolene
B) Fat emulsion
C) Flumazenil
D) Naloxone
E) Propofol
The correct response is Option B.
This patient is experiencing symptoms of lidocaine toxicity. Lidocaine toxicity occurs within a few minutes
after injection but can occur up to 60 minutes after injection. The maximum dose of lidocaine without
epinephrine is 4.5 mg/kg and with epinephrine is 7 mg/kg. Symptoms of lidocaine toxicity can range from
central nervous system (CNS) excitement (circumoral/tongue numbness, metallic taste, lightheadedness,
dizziness, visual and auditory disturbances, disorientation, drowsiness), and at higher
doses CNS depression (muscle twitching, convulsions, unconsciousness, coma, respiratory depression
and arrest, cardiovascular depression and collapse). Cardiovascular manifestations include chest pain,
shortness of breath, palpitations, hypotension, and syncope. Of the options presented, fat emulsion
(Intralipid) is the treatment of choice. Flumazenil is the treatment for benzodiazepine overdose, naloxone
is for opioid overdose, dantrolene is for malignant hyperthermia, and propofol is used for induction and
maintenance of general anesthesia.
Reference(s)
1. Mustoe TA, Buck II DW, Lalonde DH. The safe management of anesthesia, sedation, and pain in
plastic surgery. Plast Reconstr Surg. 126:165e, 2010.
2. Failey C, Aburto J, Garza de la Portilla H, et. al. Office-based outpatient plastic surgery utilizing total
intravenous anesthesia. Aesth Surg J. 33(2) 270-274, 2013.
A 25-year-old right-hand–dominant woman sustains a full-thickness circumferential burn to the right
upper extremity from the shoulder to the wrist. She undergoes early excision and grafting. Six months
after treatment, she undergoes operative release of a severe flexion contracture of the elbow (greater
than 50% loss of joint motion), resulting in a large defect. Which of the following is the most appropriate
option for reconstruction of the defect?
A) Free fasciocutaneous flap
B) Full-thickness skin grafting
C) Local perforator flap
D) Split-thickness skin grafting
E) Z-plasty
The correct response is Option A.
In severe burn scar contractures, adjacent tissue transfer (Z-plasty, VY-plasty) and skin grafts are not
indicated. Perforator-based local flaps have low recurrence rates but one limitation of this technique is
the availability of local normal skin. In this specific case, no normal skin is available. Free tissue transfer
is the best option. Perforator vessels are normally protected and can serve as recipient vessels for the
free flap transfer.
Reference(s)
1. Hayashida K, Akiti S. Surgical treatment algorithms for post-burn contractures. Burns Trauma. 2017;
5:9.
2. Stekelenburg CM, Marck RE, Tuinebreijer WE, et al. A systemic review on burn scar contracture
treatment: searching for evidence. J Burn Care Res. 2015 May-Jun;36(3):e153-61.
An otherwise healthy 62-year-old woman presents with mild edema, some hemosiderin deposition, and a
clean, shallow, painful ulcer about 2 cm in size above the left medial malleolus. Medical history includes a
left ankle fracture 15 years ago. She does not smoke cigarettes. She has a job which requires that she
stand for 8-hour shifts. Distal pulses are present and ankle brachial index is .94. Which of the following is
the most appropriate initial management?
A) Debride the wound and apply a split-thickness skin graft
B) Elevate and apply serial compression dressings (Unna boot)
C) Hyperbaric oxygen therapy (HBOT)
D) Optimize the wound bed with bilaminate neodermis (Integra)
E) Strip the greater saphenous vein and ligate the perforators
The correct response is Option B.
Venous insufficiency is staged using the CEAP (clinical, etiologic, anatomical, and pathophysiologic)
classification. The patient presented in this scenario meets the criteria for C6 (Clinical 6) criteria with the
presence of an active ulcer. Compression and keeping the wound clean are the initial, primary, and
mainstay therapies for healing venous ulcers. The only option listed that provides compression and
wound care is to clean the wound, elevate, and apply serial compression dressings (Unna boot). After a
trial of compression and wound bed optimization, closure can be considered. The literature does not
provide conclusive evidence that skin grafting is a superior or desired closure. There are studies that
demonstrate the superiority of Apligraf in achieving wound closure. If the perforators are found to be the
source of the issue, ligation may reduce the recurrence of ulcers in the area but studies comparing
ligation and wound care do not show earlier closure of ulcers present. Hyperbaric oxygen therapy
(HBOT) is not indicated in this situation.
Reference(s)
1. Ratliff CR, Yates S, McNichol L, Gray M. Compression for Primary Prevention, Treatment, and
Prevention of Recurrence of Venous Leg Ulcers: An Evidence-and Consensus-Based Algorithm for Care
Across the Continuum. J Wound Ostomy Continence Nurs. 2016 Jul-Aug;43(4):347-64.
2. Game FL, Jeffcoate WJ. Dressing and Diabetic Foot Ulcers: A Current Review of the Evidence. Plast
Reconstr Surg. 2016 Sep;138(3 Suppl):158S-64S.
3. Pascarella L, Shortell CK. Medical management of venous ulcers. Semin Vasc Surg. 2015
Mar;28(1):21-8.
A 63-year-old woman has a superficial surgical site infection at her abdominal incision two weeks
following deep inferior epigastric perforator (DIEP) breast reconstruction. The CDC categorizes a
“superficial incisional surgical-site infection” as occurring within how many days of the procedure?
A) No more than 7 days
B) No more than 14 days
C) No more than 30 days
D) No more than 60 days
E) No more than 90 days
The correct response is Option C.
Superficial incisional surgical-site infection is defined by the following criteria:
Date of event for infection occurs within 30 days of a procedure AND involves only skin and
subcutaneous tissue of the incision AND the patient has at least one of the following:
A. purulent drainage at incision site
B. positive cultures from the incisions or underlying tissue
C. superficial incision that is deliberately opened by a surgeon or other designee with the following signs
or symptoms: pain or tenderness; localized swelling; erythema; or heat
D. diagnosis by the surgeon or attending physician designee
Reference(s)
1. Centers for Disease Control and Prevention. Surgical Site Infection (SSI) Event. CDC Procedure
Assisted Module; January 2016. http://www.cdc.gov/nhsn/pdfs/pscmanual/9pscssicurrent.pdf. Date
Accessed: Jan 2018.
2. Institute for Healthcare Improvement. How-to Guide: Prevent Surgical Site Infections. 2012,
Cambridge, MA.
http://www.ihi.org/resources/Pages/Tools/HowtoGuidePreventSurgicalSiteInfection.aspx. Date Accessed:
2012
52-year-old man presents with a chronic ulcer of the lower extremity. Current medications include
prednisone for management of rheumatoid arthritis. In addition to standard local wound care, which of the
following treatments is most appropriate?
A) Folate
B) Hyperbaric oxygen therapy (HBOT)
C) Long-acting insulin
D) Vitamin A
E) Vitamin C
The correct response is Option D.
Malnutrition is a well-established risk factor for the development of chronic wounds. Vitamin A has been
shown in multiple studies to offset the detrimental effects of corticosteroids on wound healing.
Appropriate glucose management is critical to the treatment of diabetic ulcers, but insulin would not be
indicated in the absence of uncontrolled diabetes mellitus.
Vitamin C is required as a cosubstrate for enzymes involved in collagen production, and its
supplementation is recommended for the nutritionally deficient. However, vitamin C has not been shown
to be of any benefit to wound healing in the setting of chronic corticosteroid therapy per se.
Hyperbaric oxygen therapy delivers oxygen to tissues by both hemoglobin-dependent transport and
vastly increased dissolved oxygen content in blood when a patient breathes 100% oxygen at pressures
well above atmospheric level. This improves wound healing by multiple cellular mechanisms in select
wounds. However, benefits in treatment of chronic corticosteroid use have not been demonstrated.
Elevated serum homocysteine has been associated in multiple studies with impaired wound healing and
increased risk of coronary and cerebrovascular disease due to its enhancement of clotting
pathways. Folate supplementation is often used to treat hyperhomocysteinemia, but conclusive benefit in
chronic wounds is not as well-established.
Reference(s)
1. Molnar JA, Vlad LG, Gumus T. Nutrition and chronic wounds: improving clinical outcomes. Plast
Reconstr Surg. 2016 Sep;138(3 Suppl):71S-81S.
2. Wang AS, Armstrong EJ, Armstrong AW. Corticosteroids and wound healing: clinical considerations in
the perioperative period. Am J Surg. 2013;206:410-7.
A 45-year-old woman comes to the office for consultation about immediate bilateral breast reconstruction
of a right-sided tumor measuring 2.5 cm. Biopsy reveals a HER-2/neu negative invasive ductal carcinoma
without lymphovascular invasion. The patient requests nipple-sparing mastectomy. Physical examination
shows a palpable mass is located in the right upper outer quadrant approximately 1.5 cm from the nippleareola
complex and is freely mobile. There is no lymphadenopathy on exam. Based on current literature,
which of the following best describes this patient’s candidacy for the requested procedure?
A) Good candidate based on current presentation
B) Not a candidate because of lymph node status
C) Not a candidate because of tumor location
D) Not a candidate because of tumor pathology
E) Not a candidate because of tumor size
The correct response is Option C.
Nipple-sparing mastectomy or total skin-sparing mastectomy is becoming an increasingly popular choice
for women because of the excellent cosmetic outcomes and the ability to save the nipple-areola complex
that may provide psychological benefits with increased patient satisfaction as well. Nipple-sparing
mastectomy appears to be oncologically safe with low risks of cancer recurrence in the literature thus
far. However, there has been little long-term follow-up, so this approach is still somewhat controversial
because the oncologic safety and locoregional recurrence have not been examined definitively. Although
certain centers are pushing the envelope regarding the use of this technique in a wide range of patients,
the current literature supports the following exclusion criteria:
A. Tumor size greater than 5 cm
B. Tumor location less than 2 cm from the nipple
C. Evidence of axillary disease
D. Tumor involvement on retroareolar biopsy
E. Lymphovascular invasion, human epidermal growth factor receptor-2 positivity, and/or HER-2/neu
positivity on biopsy
The current patient’s tumor was found to have a tumor-to-nipple distance of 1.5 cm which is a relative contraindication to nipple-sparing mastectomy in this case.
Reference(s)
1. Headon HL, Kasem A, Mokbel K. The Oncological Safety of Nipple-Sparing Mastectomy: A Systematic
Review of the Literature with a Pooled Analysis of 12,358 Procedures. Arch Plast Surg. 2016 Jul; 43(4):
328-338.
2. Mallon P, Feron JG, Couturaud B, et al. The Role of Nipple-sparing Mastectomy in Breast Cancer: A
Comprehensive Review of the Literature. Plast Reconstr Surg. 2013 May;131(5):969-84.
Which of the following factors has been shown to have the greatest impact on infection reduction in
trauma patients with a lower-extremity open fracture?
A) Definitive irrigation and debridement, and initiation of negative pressure wound therapy within 12 hours of injury
B) Early intramedullary reaming and nail fixation
C) Evaluation of the patient at a level I trauma center
D) Initial debridement of the wound by a senior surgeon
E) Intravenous administration of an antibiotic within 3 hours of injury
The correct response is Option E.
Early administration of antibiotics (less than 3 hours after injury) has been shown to be the most
important determinant of infection prevention after traumatic open fractures of the lower extremity; this is
more influential than other factors including time to initial washout, seniority of surgeon involved, as well
as severity of the extremity trauma. Guidelines differ by institution but at least a cephalosporin is
advocated with consideration given to additional gram-negative plus or minus anaerobic coverage in
grossly contaminated wounds.
In a large multi-institutional study of open fractures, Pollak et al, showed a significant decrease in
infection rate with either early direct admission (<2 hours) or transfer (<11 hours) to a level I trauma
center, though this was only true for the Gustilo Type III open tibial fracture subgroup and not all open
fractures. The authors hypothesized that early transfer resulted in earlier administration of antibiotics, though this was not directly compared.
While prompt debridement is important in obtaining wound control in traumatic lower-extremity open
fractures, no clear advantage has been shown to debridement within 6 hours versus 24 hours, with the
accepted standard that this can typically wait until daylight hours in the setting of timely prophylactic
antibiotic administration.
Intramedullary reaming has not been shown to decrease infection risk in open tibial fractures, although
early skeletal reduction and stability will minimize ongoing soft tissue damage.
Reference(s)
1. de Mestral C, Sharma S, Haas B, et al. A contemporary analysis of the management of the mangled
lower extremity. J Trauma Acute Care Surg. 2013;74(2):597-603.
2. Gosselin RA, Roberts I, Gillespie WJ. Antibiotics for preventing infection in open limb fractures.
Cochrane Database Syst Rev. 2004(1):CD003764.
3. Park JJ, Campbell KA, Mercuri JJ, Tejwani NC. Updates in the management of orthopedic soft-tissue
injuries associated with lower extremity trauma. Am J Orthop. 2012;41(2):E27-35.
A 16-year-old transmasculine (female-to-male) patient is evaluated for bilateralmastectomy forgender
confirmation. According to the World Professional Association for Transgender Health (WPATH)
Standard of Care Guidelines, this procedure is considered to be medically necessary, appropriate, and
indicated when which of the following conditions is met?
A) The patient has the capacity to make a fully informed decision
B) The patient has had 6 months of continuous hormone therapy
C) The patient has one physician who can document gender dysphoria
D) The patient has recently started pharmacologic therapy for depression
E) The patient must be at least 21 years of age
The correct response is Option A.
The criteria among most insurance plans for coverage of procedures for treatment of gender dysphoria
are quite stringent and rely on the concept of medical necessity for the patient.
According to the World Professional Association for Transgender Health (WPATH), which is considered the authority on
transgender health, standard-of-care guidelines to verify medically necessary procedures include:
–the capacity of the patient to make a fully informed decision and provide consent (patients who are
younger than 18 may provide assent, along with parental consent, for mastectomy)
–at least 12 months of hormone therapy consistent with the individual’s gender goals
–living life fully in the role of the desired sex for at least 12 months
–psychiatric illnesses must be stable and well-controlled
–documentation of gender dysphoria and the potential benefit from surgery by at least two health-care
providers
Reference(s)
1. CPT corner: Coding for sex-reassignment surgery is evolving. Plastic Surgery News, March 2015,
page 14.
2. CMS.gov. Gender Reassignment Surgery Model NCD. Page 6.
https://www.cms.gov/medicare/coverage/determinationProcess/downloads/Kalra_comment_01022016.p
df. Accessed May 25, 2018.
3. Standards of Care for the Health of Transsexual, Transgender, and Gender Nonconforming People. 7
th ed. The World Professional Association for Transgender Health, 2011.
A 25-year-old woman comes to the office for excision of a skin lesion with local anesthesia. Medical
history includes multiple food and drug allergies. Shortly after the procedure is initiated, the patient
reports itchy eyes and runny nose and becomes very anxious. Within the next 5 minutes, the patient
develops diffuse skin erythema and shortness of breath, which progresses to respiratory stridor.
Intravenous access has not been established. Intramuscular administration of which of the following
drugs is the most appropriate next step in management?
A) Diphenhydramine
B) Epinephrine
C) Ketamine
D) Midazolam
E) Triamcinolone
The correct response is Option B.
Epinephrine is the first line of treatment for patients with anaphylaxis and should be administered
intramuscularly (anterolateral thigh) as soon as the diagnosis is made. In a setting where an exact dose
can be drawn up, the recommended dosage is 0.01 mg/kg (maximum dose of 0.5 mg), administered
intramuscularly every 5 to 15 minutes as necessary to control symptoms. It can also be administered
more frequently than every 5 minutes if needed.
Greater emphasis has been placed on early administration of epinephrine in the management of
anaphylaxis. The mean time to cardiac or respiratory arrest from medication-induced anaphylaxis in a
hospital setting has been shown in one study to be only 5 minutes, with only 14% of patients receiving
epinephrine prior to arrest.
Besides intramuscular epinephrine, recommended immediate interventions for anaphylaxis include an
assessment of airway, breathing, and circulation; intravenous access; supplemental oxygen; monitoring;
and placing the patient in supine position. Depending on the initial response, other interventions include
establishing an airway, rapid intravenous fluid infusion, intravenous epinephrine infusion, bronchodilators,
steroids, H1/H2 antihistamines, and glucagon.
Ketamine and midazolam (versed) are not indicated for the treatment of anaphylaxis. Diphenhydramine
(Benadryl) and other antihistamine drugs are recommended as a second line of therapy for anaphylaxis.
The same applies to systemic corticosteroids.
Reference(s)
1. Kemp SF, Lockey RF, Simons FE, et al. Epinephrine: the drug of choice for anaphylaxis - a statement
of the world allergy organization. Allergy. 2008 Aug;63(8):1061-70.
2. Campbell RL, Li JT, Nicklas RA, et al. Emergency department diagnosis and treatment of anaphylaxis: a practice parameter. Ann Allergy Asthma Immunol. 2014 Dec;113(6):599-608.
A 2-year-old boy is brought to the office for evaluation of unilateral blepharoptosis. Examination shows a
2-mm eyelid ptosis of the right eye with 7-mm of upper eyelid excursion without visual obstruction. The
left side shows no abnormalities. Which of the following is the most appropriate treatment in this patient?
A) Frontalis suspension
B) Lenticular skin excision
C) Levator palpebrae advancement
D) Tarsoconjunctival Müllerectomy (Fasanella-Servat procedure)
E) Observation and re-evaluation at age 3
The correct response is Option E.
Correction of mild to moderate eyelid ptosis in children should be delayed until the child can cooperate
with the preoperative assessment and post operative care. This would not be reliably possible for a 2-
year-old patient. Intervention before age 3 should be considered if there is significant obstruction of the
visual axis. Levator advancement provides appropriate correction in pediatric patients with fair to good
levator function. Frontalis suspension is generally reserved for instances when levator function is poor
(less than 4 mm). Lenticular skin excision will have no reliable effect on lid position.
Reference(s)
1. Harvey D J, Iamphongsai S, Gosain A K. Unilateral congenital blepharoptosis repair by anterior levator
advancement and resection: an educational review. Plast Reconstr Surg. 2010;126(4):1325-1331.
2. Hornblass A. Eyelids. In: Oculoplastic, Orbital and Reconstructive Surgery. Vol 1. 1988:121.
A 37-year-old woman is brought to the operating room after sustaining a crush injury to the leftupper arm
during a rollover motor vehicle collision that included prolonged extraction from the vehicle. In the
operating room, the patient underwent bypass grafting to reconstruct the brachial artery after fixation of
the humerus. Postoperatively, the patient reports increasing pain of the left forearm with increasing pain
control requirement. Doppler shows intact radial and ulnar pulses. Which of the following clinical studies is most likely to help determine the treatment plan at this time?
A) Assessment of capillary refill of the finger tips
B) Doppler examination of the digital arteries
C) Duplex scan to check patency of graft
D) Measurement of the compartment pressures of the forearm
E) Pulse oximetry of the digits
The correct response is Option D.
In this scenario, the physician should rule out compartment syndrome for several reasons: the crush
injury, the reperfusion state, and pain unrelieved by pain medications prior to providing more pain relief.
Pain that is out of proportion to the expected level or out of proportion to examination findings should
alert the clinician to the possibility of compartment syndrome.
The only study option provided that would give the clinician the ability to rule out compartment syndrome
is the direct measurement of compartment pressures, which is recommended by several authors. Loss of
peripheral artery pulses or perfusion to the distal skin would be very late presentations of the ischemic
process, at a point where intervention, such as fasciotomy may not be effective. Missing this diagnosis in
a patient such as this one, may lead to loss of muscular function (ischemic muscle contracture) even if
the limb as a whole is salvaged.
The presence of palpable pulses is reassuring evidence for the patency of the bypass graft but does not
tell the clinician about the perfusion in the capillary beds of the muscle. Similarly, perfusion of the skin
and the digits does not guarantee flow in the muscle that was reperfused.
Reference(s)
1. Kistler J, Ilyas A, Thoder JJ. Forearm Compartment Syndrome Evaluation and Management. Hand
Clin. 2018 Feb;34(1):53-60.
2. Prasarn ML, Ouellette EA. Acute compartment syndrome of the upper extremity. J Am Acad Orthop
Surg. 2011 Jan;19(1):49-58.
Which of the following must be present in order to initiate a brain death examination?
A) Absence of deep tendon reflexes
B) Anoxia
C) Cause of brain death
D) Hypothermia
E) Ventilatory dependence with muscle relaxation (neuromuscular blockade)
The correct response is Option C.
Brain death is a permanent and irreversible state. There will be no return of cerebral or cortical function.
Hypothermia is not required for brain death examination; it needs to be reversed for at least 4 hours (up
to 24 hours) to establish brain death.
Neuromuscular blockade must be reversed and patient must have normal peripheral muscle response to
peripheral nerve stimulus in order to undergo brain death examination. Brain death examination includes
elicitation of brain reflexes, which can be muted by neuromuscular blockade.
Anoxic brain injury must be observed for at least 24 hours prior to beginning brain death examination.
Cause or reason for brain death must be established prior to beginning brain death examination.
Absence of deep tendon reflexes is not required to initiate the examination.
Reference(s)
1. Wijdicks EF, Varelas PN, Gronseth GS, Greer DM. Evidence-based guideline update: determining
brain death in adults: report of the Quality Standards Subcommittee of the American Academy of
Neurology. American Academy of Neurology. Neurology. 2010;74(23):1911.
2. Practice parameters for determining brain death in adults (summary statement). The Quality Standards Subcommittee of the American Academy of Neurology. Neurology. 1995;45(5):1012.
A 56-year-old man is scheduled to undergo excision of a lower extremity melanoma during regional
anesthesia. Current medications include lisinopril and occasional motrin. He does not smoke cigarettes.
Which of the following factors increases the risk of postoperative nausea and vomiting in this patient?
A) Age over 50 years
B) Male gender
C) Nonsmoking status
D) Use of anti-inflammatory medications
E) Use of local anesthetic
The correct response is Option C.
Risk factors for postoperative nausea and vomiting fall into four categories: patient-related, anesthesia-related,
surgery-related, and other factors.
Patient-related predictors are: female sex, non-smoking status, history of postoperative nausea and
vomiting/motion sickness, genetics, age of 50 years or younger, and obesity (BMI greater than 30
kg/m2).
Anesthesia-related predictors are: postoperative opioids, inhalational anesthetics, and nitrous oxide.
Surgery-related predictors are: surgery duration and surgery type.
Other factors including high patient anxiety and postoperative pain.
Reference(s)
1. Manahan MA, Johnson DJ, Gutowski KA, et al. Postoperative Nausea and Vomiting with Plastic
Surgery: A Practical Advisory to Etiology, Impact, and Treatment. Plast Reconstr Surg. 2018
Jan;141(1):214-222.
A 30-year-old woman with a history of Crohn disease undergoes ventral hernia repair. BMI is 31 kg/m2.
Which of the following is the greatest risk factor for postoperative nausea and vomiting in this patient?
A) Elevated body mass index
B) Perioperative fasting
C) Placement of nasogastric tube
D) Supplemental oxygen
E) Young age
The correct response is Option E.
Postoperative nausea and vomiting remains a significant problem after cosmetic and reconstructive
plastic surgery. Reported studies on the condition list incidences as high as 56%, whereas a metaanalysis
found that the overall incidence was 28.3%.
For many plastic surgery procedures, general inhalational anesthesia and narcotic pain control are
required and may predispose patients to postoperative nausea and vomiting (PONV).The effects can be
disastrous including hematoma, incisional dehiscence, respiratory compromise, pain, longer hospital
stay, slower recuperation, and patient dissatisfaction.
A number of risk factors have been associated with a positive overall incidence of PONV. Patient-specific
risk factors for PONV in adults include female sex, a history of PONV, non-smoking status, young age,
general versus regional anesthesia, postoperative opioids, and type of surgery.
obesity, and supplemental oxygen.
Reference(s)
1. Gan TJ, Diemuncsh P, Habib AS, et al. Consensus guidelines for the management of postoperative
nausea and vomiting. Anesth Analg. 2014 Jan;118(1):85-113.
2. Lee YZ, Lee RQ, Thinn KK, et al. How patients fare after anaesthesia for elective surgery: a survey of
postoperative nausea and vomiting, pain and confusion. Singapore Med J. 2015 Jan;56(1):40-6.
3. Manahan MA, Johnson DJ, Gutowski KA, et al. Postoperative nausea and vomiting with plastic
surgery: a practical advisory to etiology, impact, and treatment. Plast Reconstr Surg. 2018
Jan;141(1):214-222.
Which of the following scenarios represents a medical “near miss” event?
A) A patient consented for a right carpal tunnel release is surgically prepped for a left carpal tunnel release
B) A patient describes breast firmness and asymmetry three months after implant augmentation
C) A patient develops an asymptomatic pneumothorax after central venous catheter placement
D) A patient prescribed hydroxyzine is treated with hydralazine
E) A patient with a penicillin antibiotic allergy is treated with a cephalosporin antibiotic
The correct response is Option A.
This patient has the potential to suffer a wrong site surgery if time-out protocols were not
established. The surgical preparation error is a near miss. A “near miss” is an unplanned event that does
not result in injury, illness, or damage, but has the potential to do so.
It is within the standard of care to treat patients who report penicillin antibiotic allergies with a
cephalosporin antibiotic. Patient with a true penicillin allergy have about a 10% cross-reactivity with cephalosporin antibiotics. Treating a patient prescribed hydroxyzine with hydralazine is a look-alike,
sound-alike medication error. Pneumothorax after central venous catheter placement is a complication of
the procedure. Breast implant contracture is an inherent risk of breast implant augmentation.
Reference(s)
1. Haugen AS, Murugesh S, Haaverstad R, Eide GE, Søfteland E. A survey of surgical team members’
perceptions of near misses and attitudes towards Time Out protocols. BMC Surg. 2013 Oct 9;13:46.
1-year-old boy presents with a 3-cm mass on the cheek. Which of the following characteristics is most
likely to support a diagnosis of lymphatic malformation in this patient?
A) Firm and rubbery to palpation
B) History of recurrent infections
C) Presence of a bruit
D) Presence of calcified phleboliths
E) Rapid growth followed by involution
The correct response is Option B.
Lymphatic malformations (LM) are benign masses of abnormal lymphatic vessels. There are many
historic classifications, such as microcystic and macrocystic, as well as historic terms for specific areas affected, such as cystic hygroma, found in the head and neck. Cystic hygromas may lead to airway
compromise in the newborn. LM often fluctuate in size and are frequently associated with recurrent bouts
of infection. They tend to be soft and compressible on palpation, not firm and rubbery. LM are present at
birth, although they might not become evident until a little later in life, especially in the lower extremities.
However, they do not follow the growth pattern of hemangiomas, specifically rapid growth (proliferation)
followed by a period of involution. Treatment of LM may include observation for asymptomatic lesions,
surgical resection, sclerotherapy for larger cystic structures, or laser treatment for small, superficial
lesions. More recently, a mutation in the PIK3CA gene, which affects a tyrosine kinase cellular signaling
pathway, has been linked to lymphatic malformations.
Further elucidation of this genetic link may lead to improved understanding and directed treatments in the
future.
The presence of phleboliths is a common finding in venous malformations.
The presence of a bruit suggests the turbulent flow of an arteriovenous malformation.
Reference(s)
1. Bentz ML, Bauer BS, Zuker RM. Principles &Practice of Pediatric Plastic Surgery. St. Louis, MO:
Quality Medical Publishing; 2008.
A plastic surgery intern is reviewing patient’s charts for the week’s upcoming surgical cases on her
service. She realizes that a 55-year-old immediate breast reconstruction patient with diabetes and
hypertension, scheduled for surgery in 4 days, was not scheduled for an anesthesia preoperative
evaluation and, therefore, had no laboratory studies or electrocardiography scheduled. The intern informs
the attending physicians, schedules the appointment, and then calls the patient. She and the scheduler
then implement a system by which all patients age 50 years or older who have pre-existing medical
problems get an anesthesia preoperative evaluation. This is an example of which of the following core
competencies?
A) Interpersonal and communication skills
B) Medical knowledge
C) Patient care and procedural skills
D) Professionalism
E) Systems-based practice
The correct response is Option E.
This is an example of systems-based practice. According to the Accreditation Council of Graduate
Medical Education (ACGME) milestones, system-based practice subcompetencies include the ability to:
1. Work effectively in various health care delivery settings and systems relevant to their clinical specialty,
2. Coordinate patient care within the health care system relevant to their clinical specialty,
3. Incorporate considerations of cost awareness and risk/benefit analysis in patient care,
4. Advocate for quality patient care and optimal patient care systems,
5. Work in interprofessional teams to enhance patient safety and improve patient care quality,
6. Participate in identifying systems errors and in implementing potential systems solutions.
In this scenario, the intern is demonstrating the ability to “coordinate patient care within the health care
system relevant to their clinical speciality” and “participate in identifying systems errors and in
implementing potential systems solutions.” The other core competencies are not as relevant as systemsbased
practice in this scenario.
Reference(s)
1. Dinchen J, Deslauriers J, Kamran SC, et al. Milestones guidebook for residents and fellows.
Accreditation Council for Graduate Medical Education website.
https://www.acgme.org/Portals/0/PDFs/Milestones/MilestonesGuidebookforResidentsFellows.pdf.
Updated June 2017. Accessed February 27, 2019.
2. Parikh RP1, Snyder-Warwick A, Naidoo S, et al. Impact of an Event Reporting System on Resident
Complication Reporting in Plastic Surgery Training: Addressing an ACGME and Plastic Surgery
Milestone Project Core Competency. Plast Reconstr Surg. 2017 Nov;140(5):736e-745e.
A 16-year-old girl, who was born with a complete unilateral cleft of the lip, alveolus, and palate, is missing
the lateral incisor within the cleft. After secondary bone grafting of the alveolar cleft, which of the following
prosthetic treatments is the best option for dental restoration in this patient?
A) Nasoalveolar molding
B) Osseointegrated implant and crown
C) Palatal obturator
D) Removable partial denture
E) Three-unit fixed partial denture
The correct response is Option B.
Patients with cleft lip and palate frequently have absence of teeth in the alveolar cleft or teeth that may be
grossly abnormal or that erupt at an inappropriate angle and require removal. The lateral incisors are
most commonly affected, although central incisors and canines may also be affected. Alveolar bone
grafting during the mixed dentition phase restores adequate bone support for subsequent placement of
an endosseous titanium implant, to which a permanent crown may then be attached. Many studies have
reported the efficacy and safety of this approach. This has become the dentofacial prosthetic treatment of
choice for the replacement of a single tooth due to its appearance, functionality, and longevity. A
removable partial denture is one that rests on the surrounding soft tissues of the alveolar ridge and
palate. Although aesthetics may be reasonable, it may cause irritation of the surrounding soft tissues and
may produce movement during function. It is often a temporary solution at best.
A three-unit fixed partial denture is a prosthesis which spans the gap produced by the missing tooth by
anchoring to the adjacent two teeth. However, the abutment teeth often require reduction to permit
fixation of the prosthesis. While certainly longer lasting than a removable partial denture, a fixed partial
denture will need to be replaced periodically, and therefore would not be the best choice for this young
patient.
A palatal obturator is a prosthesis used to treat a residual oronasal fistula by physically blocking air
escape during speech. This decreases hypernasality. It rests on the soft tissues of the palate and may anchor to the alveolar ridge or teeth. It does not, however, play a role in dental restoration. Nasoalveolar
molding is a prosthesis-based treatment used early in life, typically for wide clefts, prior to repair of the lip
and palate, but it is not used for dental restoration.
Reference(s)
1. Bentz ML, Bauer BS, Zuker RM. Principles &Practice of Pediatric plastic Surgery. St. Louis, MO:
Quality Medical Publishing; 2008:651-2.
2. Sowmya S, Shadakshari S, Ravi MB, Ganesh S, Gujjari AK. Prosthodontic care for patients with cleft
palate. J Orofac Res. 2013;3(1):22-27.
A multi-institutional clinical trial is gathering data on the ability of a test to determine the number of
women who develop a new breast disease, and comparing this with age-matched controls. The
specificity of the test is defined as which of the following?
A) The ratio of healthy subjects diagnosed as negative and the total number of healthy patients
B) The ratio of healthy subjects diagnosed as positive and the total number of sick patients
C) The ratio of sick patients diagnosed as negative and the total number of healthy patients
D) The ratio of sick patients diagnosed as negative and the total number of sick patients
E) The ratio of sick patients diagnosed as positive and the total number of sick patients
The correct response is Option A.
The sensitivity of a test is defined as the ability of a test to correctly classify an individual as diseased
(positive in disease).
Sensitivity = a / a + c
The specificity of a test is the ability of a test to correctly classify an individual as disease free.
Specificity = d / b + d
Reference(s)
1. Kirkwood BR and Stern JAC. Essential Medical Statistics. 2nd edition. Malden, Massachusetts:
Blackwell Sciences Ltd; 2003.
A 31-year-old man presents with a posterior fracture to the body of the mandible involving the alveolus of
the first molar sustained during an assault. Open reduction and internal fixation of the fracture is
performed. Intraoperatively, the position of the mandibular first molar in the fracture prevents an
adequate reduction, and it must be extracted. Removal of how many intact tooth roots is most likely to
indicate complete extraction of the mandibular molar in this patient?
A) One
B) Two
C) Three
D) Four
The correct response is Option B.
Anatomy of the mandibular first molar is relatively consistent in that the vast majority will have two
roots. Knowledge of the number of roots is important in the setting of extraction to ensure complete
removal. If either the injury or the reduction of the mandible fracture had caused a fracture of the tooth
root itself, then complete removal of the fractured root would involve either exploration at the time of open
reduction and internal fixation or postoperative referral to an oral surgeon. A retained tooth root would
place the patient at high risk for abscess formation that could require additional treatment. Incisors,
canines, mandibular premolars, and maxillary second premolars usually have one root. Maxillary first
premolars and mandibular molars usually have two roots. Maxillary molars usually have three roots.
Reference(s)
1. Nelson SJ. Wheeler’s Dental Anatomy, Physiology, and Occlusion. 10th ed. St. Louis, MO: Elsevier
Saunders; 2015.
2. Netter FH. Atlas of Human Anatomy. 4th ed. Philadelphia, PA: Elsevier; 2006:plate 57.
A newborn is noted to have a lesion of the midline of the lower back consisting of a protruding membrane
which covers meninges, cerebrospinal fluid (CSF), and neural structures. Which of the following is the
primary goal of surgical repair?
A) Hydrocephalus mitigation
B) Increase in lower extremity strength
C) Infection prevention
D) Prevention of tethered cord syndrome
E) Restoration of bowel or bladder function
The correct response is Option C.
Meningomyelocele is the most common neural tube defect. It involves dorsal herniation of the meninges
and spinal cord through the vertebrae and may produce motor and sensory nerve deficits. It is often
diagnosed prenatally by elevated maternal serum alpha fetoprotein and ultrasonography. Treatment of
larger defects often involves both neurosurgery and plastic surgery teams. After repair of the neural placode, the goals of soft tissue reconstruction are to cover and protect the neural element, prevent
infection, and avoid any cerebrospinal fluid leak. Ideally this is performed within the first 24 to 48 hours of
life. Larger defects are often best reconstructed with muscle flaps, fasciocutaneous flaps, or a
combination of both. Many different flaps have been described, but considerations for adequate
vascularity (such as inclusion of perforator blood vessels within geometrically designed flaps) and closure
without tension are paramount.
While hydrocephalus is a common finding in patients with meningomyelocele, it is treated with
cerebrospinal fluid shunting if required.
Meningomyelocele repair does not regain or improve neural abilities that are not present at birth, such as
bowel and bladder function, and lower extremity motor and sensory function.
Symptoms related to tethering of the spinal cord may develop as the patient grows in as many as 20 to
50% of children who undergo meningomyelocele repair shortly after birth and many may require surgery
to release the scar tissue attached to the cord. However, this condition is not prevented by
meningomyelocele repair.
Reference(s)
1. Rodriguez ED. Craniofacial, head and neck surgery. In: Rodriguez ED, ed. Craniofacial, Head and
Neck Surgery and Pediatric Surgery; vol 3.
A 32-year-old man undergoes unilateral hand transplantation. Tacrolimus for immunosupression is
initiated. On routine evaluation 6 months postoperatively, a chronic increase in creatinine and a reduction
in glomerular filtration rate is noted. Which of the following is the most appropriate next step in
management?
A) Discontinuation of antihypertensive drugs
B) Discontinuation of dyslipidemia drugs
C) Initiation of oral corticosteroid therapy
D) Reduction of calcineurin inhibitor trough levels
E) Tissue biopsy of the transplanted hand
The correct response is Option D.
This patient is experiencing nephrotoxicity and chronic kidney disease (CKD) from tacrolimus.
Calcineurin inhibitor nephrotoxicity is a well-known phenomenon posttransplantation, and close
monitoring of kidney function is essential. There are multiple described pathways of kidney damage
secondary to calcineurin inhibitors including irreversible damage to all compartments of the kidney
(glomeruli, arterioles, and tubule-interstitium).
Krezdorn et al. evaluated 99 recipients of facial or extremity transplantation and concluded that kidney
dysfunction represents a major complication posttransplantation in vascularized composite allografts and
recommends pretransplant, peritransplant, and posttransplant strategies to reduce kidney damage.
These include identifying patients at risk for CKD. Pretransplantation recommendations include treating
existing renal conditions, avoiding hypotension and hypertension, limiting nephrotoxic drugs, limiting
intravenous contrast, and avoiding hypovolemia. Peritransplantation recommendations include
minimizing use of nephrotoxic agents, avoiding hypovolemia, and limiting ischemia time. Postoperative
recommendations include minimizing tacrolimus exposure including reduction of trough levels, treating
hypertension, treating hyperglycemia, treating dyslipidemia avoiding intravenous contrast, and limiting
potentially nephrotoxic drugs such as nonsteroidal anti-inflammatory drugs.
Therefore, in this patient, tacrolimus troughs should be reduced. Initiation of oral corticosteroids is not
appropriate. The patient should remain on antihypertensive and dyslipidemia drugs.
Reference(s)
1. Krezdorn N, Tasigiorgos S, Wo L, et al. Kidney dysfunction after vascularized composite
allotransplantation. Transplant Direct. 2018 Jun 1;4(7):e362.
2. Naesens M, Kuypers DRJ, Sarwal M. Calcineurin inhibitor nephrotoxicity. Clin J Am Soc Nephrol. 2009
Feb;4(2):481-508.
A 45-year-old woman with breast cancer is scheduled to undergo bilateral mastectomy. Immediate breast
reconstruction with deep inferior epigastric perforator (DIEP) flaps is planned. Which of the following
factors is most likely to increase this patient’s risk for microsurgical thrombotic complications?
A) BRCA-2 genetic mutation
B) Caprini Risk Assessment Model score of 5
C) History of prior irradiation following lumpectomy
D) History of prior thrombotic event
E) Sickle cell trait
The correct response is Option D.
Virchow recognized a triad of factors that predispose to intravascular thrombosis. These are stasis in
blood flow, endothelial (intimal) damage, and intrinsic hypercoagulability. One recent review identifies a
personal history of prior thrombotic event as perhaps the single greatest risk factor of a hypercoagulable
state. Other known hypercoagulable disorders that can be identified by specific blood test include Factor
V Leiden mutation, prothrombin gene (20210A) mutation, protein C deficiency, protein S deficiency,
antithrombin III (AT3) deficiency, lupus anticoagulant, anticardiolipin antibodies, and anti-beta-2
glycoprotein 1 antibodies. These appear to increase the risk of microsurgical thrombotic complications as
well. However, the severity of the increased risk for each remains to be fully elucidated. Another
published series of 41 patients, showed an 80% free flap success rate in patients with identified
hypercoagulable states. Therefore, a thorough preoperative evaluation of patients for microvascular
procedures may help to identify those at increased risk for thrombotic complications, guiding patient
selection and perioperative anticoagulation therapy.
The Caprini Risk Assessment Model (RAM) is used to assess venous thromboembolism (VTE) risk, and
its use has been validated in plastic and reconstructive surgery patients. The Caprini RAM score for the
patient in this question is 5 (2 risk factor points for major surgery over 45 minutes, 2 points for presence
of malignancy, and 1 point for ages 41 to 60). According to the recommendations of the American
Society of Plastic Surgeons VTE Task Force Report, one should consider postoperative
chemoprophylaxis for VTE for this patient who is at intermediate risk. However, data are lacking, which
would support the use of the Caprini Risk Assessment Model as a tool to stratify risk of microvascular
thrombotic complications.
BRCA-2 genetic mutation is a heritable condition that significantly increases lifetime risk for breast and
ovarian cancer. However, it has not been shown to play any role in risk for thrombotic events.
Similarly, sickle cell trait (heterozygous carrier of the sickle cell mutation in the hemoglobin-beta gene)
has not been shown to increase risk for microsurgical thrombotic complications.
While chest wall irradiation might negatively impact the recipient chest wall vessels for deep inferior
epigastric perforator (DIEP) flaps, the impact of radiation following a lumpectomy is very unlikely to be as
significant as that of a prior personal thrombotic event.
Reference(s)
1. Murphy RX Jr, Alderman A, Gutowski K, et al. Evidence-based practices for thromboembolism
prevention: summary of the ASPS venous thromboembolism task force report. Plast Reconstr Surg.
2012;130:168e-75e.
2. Pannucci CJ, Bailey SH, Dreszer G, et al. Validation of the Caprini risk assessment model in plastic
and reconstructive surgery patients. J Am Coll Surg. 2011;212:105-12.
3. Pannucci CJ, Kovach SJ, Cuker A. Microsurgery and the hypercoagulable state: a hematologist’s
perspective. Plast Reconstr Surg. 2015;136(4):545e-52e.
4. Wang TY, Serletti JM, Cuker A, et al. Free tissue transfer in the hypercoagulable patient: a review of
58 flaps. Plast Reconstr Surg. 2012;129:443-53.
Which of the following is the most common type/location of salivary gland cancer in the pediatric
population?
A) Acinic cell carcinoma/submandibular glands
B) Adenoid cystic carcinoma/submandibular glands
C) Carcinoma ex pleomorphic adenoma/minor salivary glands
D) Cystadenocarcinoma/parotid gland
E) Mucoepidermoid carcinoma/parotid gland
The correct response is Option E.
The most common type/location of salivary gland cancer in the pediatric population is mucoepidermoid
carcinoma of the parotid gland. There are three sites of salivary gland carcinoma in this population
including the parotid gland, submandibular glands, and minor salivary glands. In a systematic review and
meta-analysis, Zamani et al. identified the frequency of various types of salivary cancer in children
including the location and type. Their findings indicate that the most common site of salivary gland
cancers occur in the parotid gland (72%), followed by the minor salivary glands (21%) and the
submandibular glands (8%). The most common types of salivary gland cancers are mucoepidermoid
carcinoma, adenoid cystic carcinoma, and acinic cell carcinoma. In all locations, mucoepidermoid cancer
is the most common type (53% for parotid, 55% for submandibular gland, and 63% for minor salivary
glands).
Reference(s)
1. Yoshida AJ, Garcia J, Eisele DW, Chen AM. Salivary gland malignancies in children. Int J Pediatr
Otorhinolaryngol. 2014;78:174-178.
2. Zamani M, Gronhoj C, Jensen JS. Survival and characteristics of pediatric salivary gland cancer: a
systematic review and meta-analysis. Pediatr Blood Cancer. 2018;e27543.
An otherwise healthy 26-year-old woman undergoes zone 2 wide-awake flexor tendon repair of the right
index finger. A solution of 1% lidocaine with 1:100,000 epinephrine is injected into the hand and digit.
After surgical repair of the flexor digitorum profundus (FDP) tendon, the patient’s finger is still pale
without capillary refill. Administration of which of the following classes of drug is most likely to reverse the
effects of epinephrine in this patient?
A) Alpha-adrenergic receptor activator
B) Alpha-adrenergic receptor blocker
C) Beta-adrenergic receptor blocker
D) Potassium channel activator
E) Sodium channel blocker
The correct response is Option B.
The medication that is used to reverse the effects of epinephrine is phentolamine, which is an alphaadrenergic
receptor blocker. The wide-awake Hand Surgery is well described by Donald Lalonde and
utilizes the effects of local anesthesia to perform a wide variety of hand-surgical procedures without
general anesthesia.
An alpha-adrenergic receptor activator, such as epinephrine, could increase vasoconstriction and worsen
the scenario, as could a beta-adrenergic receptor blocker. Sodium channel blockers and potassium
channel blockers are not indicated for reversal of epinephrine effect.
Reference(s)
1. Higgins A, Lalonde DH, Bell M, McKee D, Lalonde JF. Avoiding flexor tendon repair rupture with
intraoperative total active movement examination. Plast Reconstr Surg. 2010 Sep;126(3):941-945.
2. Lalonde D, Bell M, Benoit P, Sparkes G, Denkler K, Chang P. A multicenter prospective study of 3,110
consecutive cases of elective epinephrine use in the fingers and hand: the Dalhousie project clinical
phase. J Hand Surg Am. 2005 Sep;30(5):1061-1067.
A 25-year-old man presents for consultation for surgical resection of a large abdominal pannus. The
patient’s mother has a history of malignant hyperthermia. Which of the following best describes the
inheritance pattern of this condition?
A) Autosomal dominant
B) Autosomal recessive
C) Mitochondrial
D) X-linked dominant
E) X-linked recessive
The correct response is Option A.
Malignant hyperthermia is an autosomal dominant trait, thus, based on Mendelian Genetics, if one parent
has a confirmed diagnosis, their biological children will have a 50% chance of inheritance of the affected
gene.
Malignant hyperthermia is a potentially life-threatening condition. Individuals with this inherited myopathy
present with a hypermetabolic reaction to potent volatile anesthetic gases, such as halothane, enflurane,
isoflurane, sevoflurane, and desflurane. Individuals are also susceptible to the depolarizing muscle
relaxant succinylcholine. The preoperative workup for an individual with suspected malignant
hyperthermia is the Caffeine-Halothane Contracture Test. In this test, a piece of skeletal muscle is
excised, and its contractile properties are determined when exposed to the ryanodine receptor agonist
halothane and/or caffeine. Abnormal contractile activity is indicative of susceptibility. Based on basic
Mendelian Genetics, the risk of inheritance is 50%.
Given this pattern of inheritance, x-linked, autosomal and mitochondrial inheritance are incorrect
answers.
Reference(s)
1. Gurunluoglu R, Swanson JA, Haeck PC, et al. Evidence-based patient safety advisory: malignant
hyperthermia. Plast Reconstr Surg. 2009;124 (Suppl.);68S-81S.
2. Riazi S, Kraeva N, Hopkins, PM. Updated guide for the management of malignant hyperthermia. Can J
Anaesth. 2018 Jun;65(6):709-721. doi: 10.1007/s12630-018-1108-0. Epub 2018 Mar 29.
A 24-year-old man who is hearing impaired requests to be evaluated for left cubital tunnel syndrome. The
patient currently lives 2 hours away, and his mother, who usually helps interpret for him, is unable to
attend the appointment. The office does not have anyone who is capable of interpreting sign language.
Which of the following is the most appropriate next step?
A) Arrange a video interpreter to be available during the appointment at the office’s expense
B) Decline to schedule an appointment because the office does not offer the language services requested
C) Help the patient arrange for an interpreter to be present at the appointment at his expense
D) Require the patient bring a friend or family member to help interpret during the appointment
The correct response is Option A.
The Americans with Disabilities Act (ADA) requires that reasonable accommodations are provided by
businesses and in public areas to allow people with disabilities to participate in daily activities. Public
places include doctors’ offices. The building and spaces should accommodate all individuals regardless
of disability. As a business and a public space, a doctor’s office must be in compliance with the ADA.
Services cannot be denied to a patient with a disability because of the disability if services could
otherwise be provided. Accommodations should be made to examine the patient with a disability as any
other.
In this case, treatment for a cubital tunnel syndrome, something offered routinely by this office, cannot be
declined based on the patient’s hearing deficit.
In this case of a patient with a hearing issue, an interpreter must be provided to help with the
appointment upon the patient’s request. This does not necessarily need to be in-person; a video
interpreter can be acceptable. The patient cannot be charged for the interpreter services whether inperson
or by video. The health care provider is expected to make a reasonable effort to provide the
service. While having the patient bring a friend or family member to the appointment would make the
appointment easier for the physician, a patient is not required to bring anyone to help interpret for
him/her.
Reference(s)
1. Evelth PA, Sherer RB. Office of compliance - fast facts. United States Congress Office of Compliance
Web site. https://www.compliance.gov/sites/default/files/wp-content/uploads/2010/03/fastfacts_ada.pdf.
Updated February 2008. Accessed February 12, 2019.
2. Hearing Loss Association of America. ADA - Americans with Disabilities Act. Hearing Loss Association
of America Web site. https://www.hearingloss.org/programs-events/advocacy/know-your-rights/ada/.
Accessed February 12, 2019.
A 48-year-old man presents to the emergency department because of spontaneous progressive pain,
swelling, cyanosis, and edema of the left lower extremity for the past 24 hours. A photograph is shown.
Medical history includes prophylactic inferior vena cava (IVC) filter placement in the setting of prolonged
immobilization secondary to traumatic closed head injury sustained 2 years ago. Physical examination
shows no dyspnea. Oxygen saturation is 98% on room air. Venous ultrasonography and CT scan show
total left deep femoral thrombosis extending into the lower IVC at the indwelling filter. Which of the
following is the most appropriate next step in management?
A) Catheter-directed thrombolysis
B) Femoral vein to IVC vascular bypass
C) Isolated extracorporeal membrane oxygenation (ECMO) support to the affected extremity
D) Open thrombectomy
E) Oral anticoagulation
The correct response is Option A.
The patient is presenting with extensive acute thrombotic occlusion resulting in clinically evident
symptomatic venous insufficiency of the extremity. If the occlusion is left untreated, progressive cyanosis
and secondary ischemia followed by gangrene develop. Locally delivered thrombolytic agents via
catheter-directed thrombolysis with or without percutaneous transluminal angioplasty is an effective first
line of treatment in this scenario where the patient presents within a few days of symptom onset (ie, prior
to clot fibrosis) and is not high-risk for bleeding. In patients who are high-risk for bleeding (eg, acute
intracerebral hemorrhage, gastrointestinal bleeding), alternative methods of restoring venous outflow
include clot retrieval through other percutaneous or open techniques (eg, transluminal aspiration
thrombectomy, open inferior vena cava (IVC) thrombectomy with or without temporizing groin
arteriovenous fistula creation). Systemic thrombolysis can be considered when other first line therapies
are not available but has been associated with high frequency of major bleeding complications in several
randomized trials (14% for streptokinase).
Systemic anticoagulation infusion helps prevent progression but does not restore acute compromised
ischemic limb secondary to venous outflow obstruction. Oral anticoagulation is not indicated for acute
management of a limb-threatening thrombosis. Femoral vein to IVC vascular bypass is not a described
procedure for venous insufficiency. Limb-threatening thrombo-occlusive venous insufficiency resulting in
a painful swollen blue leg, such as that pictured (also known as “phlegmasia cerulea dolens,” literally
“painful blue edema”) was first described with heparin-induced thrombocytopenia. It has also been
associated with cancer or life-threatening critical illness. More recently, a growing population of patients
are at risk due to unretrieved IVC filters. While IVC filter placement may protect the pulmonary vascular
bed, it does not lessen thrombotic predisposition or incidence in the lower extremities, and IVC
thrombosis with or without phlegmasia cerulea dolens has been reported to occur in 3 to 30% of patients
following IVC filter placement. Filter retrieval following its initial indicated need can lessen secondary
thrombotic complications, but data suggest that only a fraction of retrievable filters are later removed. In a
systemic review, overall retrieval was 34% with a high percentage of nonretrieval occurring for a variety
of reasons, including loss to follow up (particularly in trauma centers), limited life expectancy, and/or
unresolved underlying conditions.
Reference(s)
Test Review Report
Printed on: 2/26/2023
1. Alkhouli M, Morad M, Narins CR, Raza F, Bashir R. Inferior vena cava thrombosis. JACC Cardiovasc
Interv. 2016;9(7):629-643.
2. Angel LF, Tapson V, Galgon RE, et al. Systematic review of the use of retrievable inferior vena cava
filters. J Vasc Interv Radiol. 2011; 22:1522.
3. Fedullo PF, Roberts A. Placement of vena cava filters and their complications. UpToDate website.
https://www.uptodate.com/contents/placement-of-vena-cava-filters-and-their-complications. Updated
October 5, 2017. Accessed January 8, 2019.
4. Heilman J. Phlegmasia cerulea dolens. In: WikiMedia website.
https://commons.wikimedia.org/w/index.php?curid=49299938. Created June 7, 2016. Accessed January
8, 2019. Courtesy J Heilman.
A 54-year-old woman undergoes abdominoplasty and hysterectomy. Postoperative rivaroxaban is
initiated for prevention of deep venous thrombosis. Three days later, she presents for follow-up, and a
hematoma requiring drainage is noted. This outcome is most likely related to the fact that rivaroxaban
blocks the coagulation cascade in which of the following ways?
A) Binds factors II, VII, IX, and X
B) Prevents conversion of prothrombin to thrombin
C) Prevents degranulation of platelets
D) Prevents thrombin from activating fibrinogen
E) Prevents tissue factor:factor VIIa complex formation
The correct response is Option B.
The direct anticoagulation agent rivaroxaban (Xarelto) is a direct anticoagulant, which acts within the
clotting cascade by blocking Factor Xa, which, along with Factor Va, helps convert prothrombin to
thrombin. This step in the cascade is where the intrinsic and extrinsic pathways intersect and the
common pathway that leads to fibrinogen being cleaved to fibrin and stabilized by factor XIIIa as a crosslinked
clot.
Aspirin is a drug that interferes with platelet function. Coumadin affects the vitamin K dependent factors II
(prothombin), VII, IX and X. Heparin prevents clot propagation by blocking thrombin-mediated activation
of fibrinogen to fibrin. The primary benefits of the direct anticoagulation agents over coumadin include no
need for monitoring and equivalence in efficacy across many clinical situations. It is excreted by the
kidney, so dosages must be altered or the drug avoided in renal failure. A U.S. Food &Drug
Administration–approved reversal agent, recombinant coagulation factor Xa (Andexxa), is now available
on the market for life-threatening or uncontrolled bleeding in patients using direct anticoagulants affecting
factor Xa, like rivaroxaban. Prior to this, administration of fresh frozen plasma (FFP) was the antidote of
choice.
Reference(s)
1. Munson CF, Reid AJ. Novel oral anticoagulants in plastic surgery. J Plast Reconstr Aesthet Surg. 2016
May;69:585-593.
2. Rogers KC, Winks SW. A new option for reversing the anticoagulant effect of Factor Xa inhibitors:
Andexanet alfa (ANDEXXA). Am J Med. 2019 Jan;132:38-41.
An 85-year-old woman underwent left modified radical neck dissection for stage III metastatic cutaneous
melanoma. The procedure and immediate recovery were uneventful. On postoperative day 2, the base of
the neck was noted to be erythematous with 300 mL/24 hours output of milky fluid emanating from closed
suction drainage. A photograph is shown. Which of the following is the most appropriate next step in
management?
Figure 64-1
A) Bed rest and pressure dressing
B) Dietary modification and octreotide therapy
C) Immediate general surgery consultation for occult esophageal injury repair
D) Initiation of total parenteral nutrition
E) Laparoscopic thoracic duct ligation
The correct response is Option B.
The patient is demonstrating an iatrogenic chyle leak (CL). CL has been reported to occur in 2 to 8% of
neck dissections secondary to thoracic duct injury, which frequently goes unrecognized intraoperatively
because of the relatively low volume and clear output seen in the immobile and fasted operative
patient. Postoperatively, CL manifests as the sudden appearance of creamy white fluid drain output
associated with overlying inflammatory skin erythema following resumption of enteral feeds containing
fat. Confirmation can be made by testing drain triglyceride levels greater than serum levels, absolute
drain triglyceride level greater than 100 mg/dL, or with the presence of chylomicrons.
The majority of CL occur in the base of the left neck where the thoracic duct is largest and carries up to
75% of the body lymphatic fluid plus protein, electrolytes, and fatty acid-containing chylomicrons formed
by breakdown of long-chain fatty acids in the proximal small bowel.
Dietary modification is important to CL management. Patients commonly transition to nonfat/low-fat diets
or medium-chain fatty acid diets (which are largely water soluble and absorbed via portal venous
circulation rather than small bowel lymphatics), resulting in decreased chyle flow. Coupled with this, for
low-output (less than 500 cc/day) CL, octreotide has been shown to be a cost-effective therapy that
significantly decreases morbidity, length of stay, and need for further surgical intervention. In a study of
low-output leaks, CL typically stopped after 2 to 4 days of octreotide therapy. Octreotide is a somatostatin
analogue with a relatively long half life that inhibits chyle production by reducing gastric, pancreatic, and
intestinal secretions.
Bed rest may be helpful for CL, as activity encourages chyle return through the thoracic duct. Pressure
dressings are of equivocal efficacy and may compromise cervical skin flap perfusion. Both methods are
largely nondefinitive without concomitant dietary modification to reduce chyle flow.
Total parenteral nutrition (TPN) can also be employed to decrease chyle production by completely
bypassing the fatty acid uptake through the small bowel lymphatic system. However, TPN utilization must
be weighed against its need for central venous access, elevated risk of bacteremia, impaired wound
healing, metabolic disturbance, and high cost.
For high-output (greater than 1000 cc/day) CL or CL that fails to diminish with octreotide therapy, surgical
intervention is frequently indicated. This includes cervical reexploration although success can be variable
because of a relatively hospitable operative field secondary to local inflammation generated by
extravasated chyle. Alternatively, thoracoscopic ligation of thoracic duct has proven to be successful
intervention in high-output leaks through mass ligation of ductal tissue as it travels through the
diaphragmatic hiatus between the azygous vein and the aorta.
Iatrogenic esophageal injuries are a significant cause of morbidity and mortality. The patient is not
presenting with cervical esophageal injury, which may be associated with dysphonia, hoarseness,
cervical dysphagia, and/or subcutaneous emphysema.
There is no indication for empiric broad spectrum antibiotics in the absence of infection or in the routine
management of CL.
Reference(s)
1. Delaney SW, Shi H, Shokrani A, Sinha UK. Management of chyle leak after head and neck surgery:
review of current treatment strategies. Int J Otolaryngol. 2017;2017:8362874.
2. Ilczyszyn A, Ridha H, Durrani AJ. Management of chyle leak post neck dissection: a case report and
literature review. J Plast Reconstr Aesthet Surg. 2011;64(9):e223-e230.
3. Jain A, Singh SN, Singhal P, Sharma MP, Grover M. A prospective study on the role of octreotide in
management of chyle fistula neck. Laryngoscope. 2015;125(7):1624-1227.
4. Swanson MS, Hudson RL, Bhandari N, Sinha UK, Maceri DR, Kokot N. Use of octreotide for the
management of chyle fistula following neck dissection. JAMA Otolaryngol Head Neck Surg.
2015;141(8):723-727.
An otherwise healthy 54-year-old perimenopausal woman is scheduled for a mastectomy for biopsy-proven
right-sided grade 2 ductal carcinoma. According to the National Comprehensive Cancer Network
(NCCN) guidelines, postmastectomy radiation therapy will be the standard of care for this patient if she
has which of the following surgical outcomes?
A) 1-cm surgical margins, four positive axillary lymph nodes
B) 1-cm surgical margins, one positive axillary sentinel node
C) 1-mm surgical margins, no positive axillary nodes
D) 5-mm surgical margins, no positive axillary nodes
E) 5-mm surgical margins, three positive axillary nodes
The correct response is Option A.
Traditionally, the need for radiation therapy has been a contraindication for implant-based reconstruction,
and autologous reconstruction is the conservative gold standard for women with advanced cancer
needing postmastectomy radiation. More recently, there have been reports of successful implant
based reconstruction in the setting of postmastectomy radiation that have similar complication
profiles and good oncologic outcomes compared with autologous reconstruction. Protocols vary between
those that radiate the expander and then expand, and those that expand and then radiate the permanent
implant. Being able to anticipate which patient will require postmastectomy radiation is essential for joint
decision making about breast reconstruction with the patient prior to her mastectomy.
By National Comprehensive Cancer Network (NCCN) guidelines, relative indications for postmastectomy
radiation therapy include: positive sentinel node with unknown status of other axillary nodes, one to
three positive nodes on permanent histology, and close surgical margins (less than 5 mm).
Postmastectomy radiation is recommended as the standard of care in the situations of positive surgical
margins with the inability to get clear margins and four or more positive lymph nodes.
Reference(s)
1. Gradishar WJ, Anderson BO, Abraham J, et al. NCCN Clinical practice guidelines in oncology - breast
cancer. National Comprehensive Cancer Network Web
site.https://www.nccn.org/professionals/physician_gls/default.aspx. Updated February 8, 2019. Accessed
February 11, 2019.
2. Ho AY, Hu ZI, Mehrara BJ, Wilkins EG. Radiotherapy in the setting of breast reconstruction: types,
techniques, and timing. Lancet Oncol. 2017 Dec;18(12):e742-e753.